2014 iTE

Lakukan tugas rumah & ujian kamu dengan baik sekarang menggunakan Quizwiz!

43. You are treating an 18-year-old college freshman for allergic rhinitis. It is September, and he tells you that he has severe symptoms every autumn that impair his academic performance. He has a strongly positive family history of atopic dermatitis. Which one of the following intranasal medications is considered optimal treatment for this condition? J) Glucocorticoids K) Cromolyn sodium L) Decongestants M) Antihistamines

ANSWER: A Topical intranasal glucocorticoids are currently believed to be the most efficacious medications for the treatment of allergic rhinitis. They are far superior to oral preparations in terms of safety. Cromolyn sodium is also an effective topical agent for allergic rhinitis; however, it is more effective if started prior to the season of peak symptoms. Because of the high risk of rhinitis medicamentosa with chronic use of topical decongestants, these agents have limited usefulness in the treatment of allergic rhinitis. Azelastine, an intranasal antihistamine, is effective for controlling symptoms but can cause somnolence and a bitter taste. Oral antihistamines are not as useful for congestion as for sneezing, pruritus, and rhinorrhea. Overall, they are not as effective as topical glucocorticoids.

100. When assessing the nutritional status and growth of a full-term infant, it is useful to know that birth weight is expected to be regained within F) 5 days G) 14 days H) 21 days I) 28 days

ANSWER: B A helpful guideline for assessing normal growth of a full-term healthy infant is that birth weight should be regained within 14 days. Other useful guidelines for healthy term infants include an average weight gain of 30 grams (1 oz) per day for the first month of life and doubling of birth weight between 4 and 5 months of age.

6. A 39-year-old female presents with lower abdominal/pelvic pain. On examination, with the patient in a supine position, you palpate the tender area of her lower abdomen. When you have her raise both legs off the table while you palpate the abdomen, her pain intensifies. Which one of the following is the most likely diagnosis? F) Appendicitis G) A hematoma within the abdominal wall musculature H) Diverticulitis I) Pelvic inflammatory disease J) An ovarian cyst

ANSWER: B A reduction of the pain caused by abdominal palpation when the abdominal muscles are tightened is known as Carnett's sign. If the cause of the pain is visceral, the taut abdominal muscles may protect the locus of pain. In contrast, intensification of pain with this maneuver points to a source of pain within the abdominal wall itself.

87. A 57-year-old female on dialysis for end-stage renal disease develops chronic, severe generalized pain. Which one of the following opioids is preferred for management of her pain? J) Codeine K) Fentanyl L) Hydrocodone M) Morphine

ANSWER: B Fentanyl and methadone are the preferred opioids for use in patients with end-stage renal disease (SOR C). Fentanyl is metabolized in the liver and has no active metabolites. All of the other listed opioid medications have active metabolites that can accumulate in patients with renal failure, leading to serious side effects. These agents should be avoided in patients on dialysis.

97. An 18-month-old male with a history of prematurity at 36 weeks gestation but no baseline lung disease is brought to the emergency department with a fever of 38.3°C (100.9°F), rhinorrhea, cough, wheezing, mild tachypnea, and an oxygen saturation of 88%. A chest radiograph reveals perihilar infiltrates, and a nasal swab is positive for respiratory syncytial virus (RSV) antigen. Which one of the following management options has evidence of benefit for this patient? A) Aerosolized ribavirin B) Supplemental oxygen C) Intravenous corticosteroids D) Macrolide antibiotics

ANSWER: B Respiratory syncytial virus (RSV) bronchiolitis is responsible for approximately 2.1 million health care encounters annually in the United States. The child in this case has a typical presentation of RSV bronchiolitis. The diagnosis can be made clinically, although specific testing for RSV is often used in the hospital setting to segregate RSV-infected patients from others. Management is primarily supportive, especially including maintenance of hydration and oxygenation. Bronchodilators, corticosteroids, and antiviral agents do not have a significant impact on symptoms or the disease course. Ribavirin is not recommended for routine use due to its expense, conflicting data on effectiveness, and potential toxicity to exposed health care workers. Antibiotics are of no benefit in the absence of bacterial superinfection.

64. A 40-year-old runner complains of gradually worsening pain on the lateral aspect of his foot. He runs on asphalt, and has increased his mileage from 2 miles/day to 5 miles/day over the last 2 weeks. Palpation causes pain over the lateral fifth metatarsal. The pain is also reproduced when he jumps on the affected leg. When you ask about his shoes he tells you he bought them several years ago. Which one of the following is the most likely diagnosis? F) Ligamentous sprain of the arch G) Stress fracture H) Plantar fasciitis I) Osteoarthritis of the metatarsal joint

ANSWER: B Running injuries are primarily caused by overuse due to training errors. Runners should be instructed to increase their mileage gradually. A stress fracture causes localized tenderness and swelling in superficial bones, and the pain can be reproduced by having the patient jump on the affected leg. Plantar fasciitis causes burning pain in the heel and there is tenderness of the plantar fascia where it inserts onto the medial tubercle of the calcaneus.

4. A 15-year-old male is brought to the office for a well child visit. His parents report that he has had a nighttime cough and wheezing for the past several months. He is otherwise healthy and up-to-date on all of his immunizations. You suspect that he has asthma. Which one of the following would be most appropriate at this point? A) Treat empirically with a short-acting Beta-agonist B) Perform spirometry C) Order radiologic testing D) Start an inhaled corticosteroid E) Start a leukotriene inhibitor

ANSWER: B The American Academy of Asthma, Allergy, and Immunology recommends that asthma not be diagnosed or treated without spirometry. Once the diagnosis is confirmed, treatment should commence with a short-acting Beta-agonist as needed, followed by stepwise treatment based on the severity of asthma.

74. A 36-year-old white female calls you to report that she removed a small blood-engorged tick from her upper arm the previous evening, 3 days after returning from a camping trip in the mountains of New Hampshire. Which one of the following would you recommend? F) A single 200-mg dose of doxycycline G) Doxycycline, 100 mg twice daily for 7 days H) Azithromycin (Zithromax), 500 mg daily for 7 days I) Topical mupirocin ointment (Bactroban), twice daily for 3 days J) No treatment unless she develops the typical erythema migrans rash

ANSWER: A In areas where there is at least a 20% rate of tick infection, 200 mg of doxycycline within 72 hours of tick removal is appropriate as prophylaxis for Lyme disease if the tick is engorged or is suspected to have been attached for at least 36 hours. The Northeast and Upper Midwest are the most endemic areas. The other oral antibiotics listed are appropriate for cases of acute erythema migrans and suspected infection. Watchful waiting is not recommended, and there is no evidence for treatment with topical antibiotics alone (SOR C).

31. A 43-year-old asymptomatic male is found to have slightly elevated ALT (SGPT) and AST (SGOT) levels on laboratory work prior to donating blood. He feels well and is otherwise healthy. Which one of the following should be ordered to evaluate the patient for hereditary hemochromatosis? F) A serum iron panel, including a serum ferritin level and transferrin saturation G) An "-fetoprotein (AFP) level H) HFE genetic testing I) Hepatic ultrasonography J) A liver biopsy

ANSWER: A Initial testing with serum ferritin levels and transferrin saturation is indicated when hereditary hemochromatosis is suspected. Normal values for these tests exclude iron-mediated organ dysfunction. Genetic testing is indicated if the serum ferritin level is >300 ng/mL in men or >200 ng/mL in women, or if transferrin saturation is 345%. A liver biopsy would be indicated to determine hepatic iron content and histopathology if the ferritin level were 31000 ng/mL or liver transaminases were elevated in a patient who is homozygous for C282Y. An "-fetoprotein level and hepatic ultrasonography would be indicated to detect hepatocellular carcinoma if the condition has already advanced to cirrhosis.

11. To prevent joint damage from gout, uric acid levels should be lowered by medication to J) <6.0 mg/dL K) <8.0 mg/dL L) <10.0 mg/dL M) a level that keeps the patient symptom-free for 6 months

ANSWER: A Targets for uric acid levels in patients with gout vary according to published guidelines but range from 5 to 6 mg/dL. Patients may be symptom-free at higher levels but risk joint damage even without acute episodes (SOR A).

53. A 45-year-old male with no known medical problems sees you for a general physical examination. Which one of the following screening measures is recommended by the U.S. Preventive Services Task Force for this patient? J) A fasting lipid profile K) Colorectal cancer screening L) Abdominal aortic aneurysm screening M) Testicular examination N) Prostate-specific antigen (PSA) testing

ANSWER: A The U.S. Preventive Services Task Force recommends that all men 35 years of age and older be screened for dyslipidemia (evidence rating A). Men who are 20-35 years of age should be screened only if they have cardiovascular risk factors (evidence rating B). Prostate-specific antigen (PSA) testing should not be performed at any age because the harms outweigh the benefits (evidence rating D). Colorectal cancer screening should begin at age 50 for men of average risk (evidence rating A). Abdominal aortic aneurysm (AAA) screening once by ultrasonography is recommended for men age 65-75 with a family history of AAA, and for those who have smoked at least 100 cigarettes in their lifetime (evidence rating B). Testicular examinations, either by self-examination or clinical screening, should not be performed routinely (evidence rating D).

92. You test a patient's muscle strength and find that his maximum performance consists of the ability to move with gravity neutralized. This qualifies as which grade of muscle strength, on a scale of 0 to 5? P) 0 Q) 1 R) 2 S) 3 T) 4

ANSWER: C Muscle strength is scored on a scale of 0 to 5. The inability to contract a muscle is scored as 0. Contraction without movement constitutes grade 1 strength. Movement with the effect of gravity neutralized is grade 2 strength, while movement against gravity only is grade 3 strength. Movement against gravity plus some additional resistance indicates grade 4 strength. Normal, or grade 5, strength is demonstrated by movement against substantial resistance.

47. A 78-year-old male experiences two episodes of near-syncope within several hours. You order an EKG, which is shown below. Which one of the following does this EKG show? M) Atrial fibrillation with a slow ventricular response N) Sinus bradycardia O) Complete heart block P) 2:1 Mobitz AV block

ANSWER: C The EKG reveals P waves unrelated to the QRS complex, in that the PR interval is variable and random. With atrial fibrillation there is no P wave. Sinus bradycardia has a P wave associated with each QRS complex and a fixed PR interval. With 2:1 AV block there would be two P waves followed by a QRS. Mobitz-type blocks have a consistent PR interval, often with a widened QRS.

2. A 44-year-old male with papulopustular rosacea sees you for follow-up. You have been treating his condition with topical azelaic acid (Finacea), and although his condition is improved he is not satisfied with the results. You suggest adding which one of the following oral medications? A) Clarithromycin (Biaxin) B) Clindamycin (Cleocin) C) Doxycycline D) Erythromycin E) Metronidazole (Flagyl)

ANSWER: C The only FDA-approved oral treatment for acne rosacea is doxycycline at a subantimicrobial dosage (40 mg daily). This does not contribute to antibiotic resistance, even when used over several months, and is better tolerated than higher dosages. Other antibiotics have limited and low-quality supporting evidence of efficacy and may lead to antibiotic resistance.

30. Which one of the following is the most accurate imaging study for assessing early osteomyelitis? A) Plain radiography B) Ultrasonography C) CT D) MRI E) A bone scan

ANSWER: D Osteomyelitis is a serious complication of diabetic foot infections and is present in up to 20% of mild to moderate infections and in 50%-60% of severe infections. While a bone biopsy and/or bone cultures are definitive for making the diagnosis, radiologic studies can also be helpful. Plain radiography may show bony destruction but has a sensitivity for osteomyelitis ranging from 28% to 75%, depending on the timing of the examination and the severity of the infection. It may take weeks for these infections to become apparent on plain radiographs. The sensitivity of triple-phase technetium bone scans is up to 90% but they have low specificity for osteomyelitis. The 90% sensitivity and 80% specificity of MRI is superior to all other imaging modalities.

28. A 75-year-old male with a history of hypertension sees you after experiencing an episode of numbness on his right side and loss of strength in his right arm. The numbness and weakness resolved spontaneously within 20 minutes. Carotid Doppler ultrasonography and cerebral angiography both reveal significant carotid stenosis. In addition to starting aspirin, which one of the following would be the most appropriate next step for this patient? F) Aggressive lowering of blood pressure G) Clopidogrel (Plavix) H) Carotid artery stenting I) Evaluation for occult patent ductus arteriosus J) High-dose statin therapy

ANSWER: E Statin drugs are effective for preventing stroke, which should be the key goal in this high-risk patient. They may stabilize the intimal wall. Rapid lowering of blood pressure could cause brain injury by reducing blood flow in patients with carotid stenosis. Any evidence of hypoperfusion needs to be corrected immediately. Combination therapy with aspirin and clopidogrel is associated with an increased risk of bleeding and is not recommended for stroke prevention. Patients over age 70 have worse outcomes with carotid stenting than with endarterectomy. Occult patent ductus arteriosus has not been shown to be a significant risk factor for stroke.

23. The Infectious Diseases Society of America recommends which one of the following as the drug of choice for group A streptococcal pharyngitis? P) Azithromycin (Zithromax) Q) Cefadroxil R) Cephalexin (Keflex) S) Clindamycin (Cleocin) T) Penicillin

ANSWER: E The Infectious Diseases Society of America recommends that penicillin remain the treatment of choice for group A streptococcal pharyngitis because of its proven efficacy, safety, narrow spectrum, and low cost. Penicillin-resistant group A Streptococcus has never been documented. Amoxicillin is often used in place of penicillin V as oral therapy for young children, primarily because of acceptance of the taste of the suspension. The other options listed are all possible regimens for group A streptococcal pharyngitis but penicillin is still considered the treatment of choice.

7. Which one of the following is one of the five basic principles of the patient-centered medical home? K) Utilizing the latest research and advances in treatment and diagnosis L) Coordinating a patient's care across all elements of the health care system M) Acting as a gatekeeper to limit access to specialist care N) Serving as the base of a pyramid in support of a complex health care system O) Transitioning away from delivering care in an office, and focusing on meeting patients in their own homes

ANSWER: B The concept of a medical home was first suggested by the American Academy of Pediatrics in 1967 to describe the ideal care of children with disabilities. In 2004 the Future of Family Medicine Project adapted this concept to describe how primary care should be based on "continuous, relationship-centered, whole-system, comprehensive care for communities." In 2007 all of the major primary care organizations collaborated to define the foundational principles of the patient-centered medical home (PCMH). These principles include the following: - Comprehensiveness: Most preventive, acute, and chronic care for individual patients can be performed at the PCMH. - Patient Centered: The PCMH provides care that is relationship-based, with an orientation toward the whole person. - Coordination: The PCMH coordinates care for patients across all elements of the health care system. - Accessibility: The PCMH works to provide patients with timely access to providers. Quality: The PCMH continuously works to improve care quality and safety.

54. A 35-year-old female with a history of chronic abdominal pain and diarrhea develops tender red nodules on her shins. These findings are most consistent with which one of the following? O) Celiac disease P) Crohn's disease Q) Diverticular disease R) Irritable bowel syndrome S) Clostridium difficile colitis

ANSWER: B Unlike the other conditions listed, Crohn's disease is associated with many extragastrointestinal conditions: erythema nodosum (as with this patient), anemia, inflammatory arthropathies, uveitis, and venous thromboembolism (level of evidence 3).

52. Blood pressure classification in children is based on F) sex, weight, and height G) sex, weight, and age H) sex, height, and age I) weight, height, and age

ANSWER: C Blood pressure in children should be measured with an appropriate size cuff. Blood pressure standards are based on age, sex, and height, and provide a precise classification of blood pressure according to body size. Blood pressure tables for children now include the 50th, 90th, 95th, and 99th percentiles by age, sex, and height (SOR A).

75. A 45-year-old female had myalgias, a sore throat, and a fever 2 weeks ago. She now has anterior neck tenderness and swelling, with pain radiating up to her ears. Your examination reveals a tender goiter. Which one of the following would support a diagnosis of subacute granulomatous thyroiditis? K) Pretibial myxedema L) Exophthalmos M) Multiple nodules on ultrasonography N) Low radioactive iodine uptake (<5%)

ANSWER: D Subacute granulomatous thyroiditis is the most common cause of thyroid pain. Free T4 is elevated early in the disease, as it is in Graves disease; however, later in the disease T4 becomes depressed and then returns to normal as the disease resolves. Pretibial myxedema, exophthalmos, and a thyroid thrill or bruit can all be found in Graves disease, but are not associated with subacute granulomatous thyroiditis. Multiple nodules on ultrasonography suggests multinodular goiter rather than subacute granulomatous thyroiditis. Patients with subacute granulomatous thyroiditis will have a low radioactive iodine uptake (RAIU) at 24 hours, but patients with Graves disease will have an elevated RAIU (SOR C).

73. An 18-month-old male is brought to your office by his mother. The patient is tugging at both ears and has a temperature of 39.0°C (102.2°F). You diagnose bilateral acute otitis media for the third time in the last 6 months. The most recent infection was 3 weeks ago and resolution of the infection was documented after 10 days of treatment with amoxicillin. Which one of the following antibiotic regimens would be most appropriate at this time? A) Amoxicillin, 45 mg/kg/day for 10 days B) Amoxicillin, 90 mg/kg/day for 10 days C) Amoxicillin, 90 mg/kg/day for 10 days followed by prophylactic treatment with amoxicillin for 6 months D) Amoxicillin/clavulanate (Augmentin), 90 mg/kg/day for 10 days E) Amoxicillin/clavulanate, 90 mg/kg/day for 10 days followed by prophylactic treatment with amoxicillin for 6 months

ANSWER: D Although high-dose amoxicillin (90 mg/kg/day) is recommended as the antibiotic of choice for acute otitis media (AOM) in the nonallergic patient, amoxicillin/clavulanate is recommended if a child has received antibiotic therapy in the previous 30 days. Prophylactic antibiotics are not recommended, as harms outweigh benefits. Tympanostomy tubes are an option if a child has had three episodes of AOM in the past 6 months or four episodes in the past year with at least one episode in the past 6 months.

25. A 76-year-old male with metastatic cancer, diabetes mellitus, and stage IV chronic renal disease develops confusion and myoclonus. His current medications include enalapril (Vasotec), 10 mg/day; glipizide (Glucotrol), 10 mg/day; and morphine sulfate, 30 mg every 4 hours for pain. The morphine was started 4 weeks ago and the dosage was gradually increased until the pain was controlled. Which one of the following is the most likely cause of his symptoms? F) A drug-drug interaction G) Metastasis to the lumbar spine H) Diabetic neuropathy I) Toxic metabolites of morphine

ANSWER: D Morphine should be avoided in patients with renal insufficiency because the toxic metabolites morphine-3-glucuronide and morphine-6-glucuronide are not eliminated by the kidneys. Accumulation of these metabolites causes neuroexcitatory effects, including confusion, sedation, respiratory depression, and myoclonus. Fentanyl and methadone are considered the safest opioids to use in patients with end-stage renal disease, but they require careful titration, dosage adjustments as necessary, continued monitoring, and an awareness of possible interactions with other medications that patients may be taking.

48. A disheveled 89-year-old male with dementia who relies on a caregiver for bathing, dressing, shopping, and meal preparation is brought in for continued evaluation of weight loss. No medical cause has been found at this point. On examination a large purplish bruise is noted over his posterior leg and a more faded greenish-yellow bruise is noted over his abdomen, which his caregiver explains by saying that he has fallen several times recently. The patient is also noted to have a large sacral decubitus ulcer. Which one of the following should you suspect as the cause of bruising in this patient? A) Senile purpura B) Thrombocytopenia C) Leukemia D) Elder abuse E) Cushing syndrome

ANSWER: D This patient has numerous red flags for elder abuse, including unexplained weight loss, reliance on a caregiver, a disheveled appearance, a pressure ulcer, and bruising in locations that are not typically associated with unintentional trauma from falls. Although the other listed causes of bruising are possible, in this scenario the index of suspicion should be highest for elder abuse.

76. A 48-year-old female sees you for routine follow-up. She was diagnosed with type 2 diabetes mellitus 2 years ago and has been treated with metformin (Glucophage), 850 mg orally 3 times daily, and glipizide (Glucotrol XL), 20 mg orally daily, along with diet and exercise. Her other medical problems include hypertension and obesity. She has no known cardiovascular disease or microvascular complications. She came in for laboratory testing yesterday, and her hemoglobin A1c is 8.0% (N <5.7%). Which one of the following medications would help with both glycemic control and weight loss for this patient? A) Exenatide (Byetta) B) Pioglitazone (Actos) C) Sitagliptin (Januvia) D) Insulin

ANSWER: A Given the information about this patient, such as her relatively recent diagnosis, her age, and her lack of macro- or microvascular complications, a more strict hemoglobin A1c goal is indicated. There are several oral and injectable medicines that are reasonable choices in this case. Exenatide is an injectable GLP-1 agonist that is associated with weight loss. Pioglitazone is also effective but is associated with fluid retention rather than weight loss. Sitagliptin is a dipeptidyl peptidase IV (DPP-IV) inhibitor that may be a reasonable option in this case, but is not associated with weight loss. Insulin, either basal only, mixed, or basal-bolus regimens, may also be the best option for the patient described, but it does cause weight gain. Cost is another major consideration in treatment decisions, but more information would be needed to address this issue.

50. A 25-year-old female with hypothyroidism sees you for preconception counseling. Her thyroid problem has been well managed with levothyroxine (Synthroid), 75 :g daily, but she asks your advice about changing her treatment to something more natural now that she is planning to become pregnant. Which one of the following is the best recommendation for this patient? K) Continue the current dosage of levothyroxine L) Reduce the current dosage of levothyroxine to 50 :g daily M) Change to a comparable dosage of combination levothyroxine/L-triiodothyronine N) Change to a comparable dosage of desiccated thyroid

ANSWER: A Untreated hypothyroidism during pregnancy impairs fetal development and increases the risk of spontaneous miscarriage, prematurity, preeclampsia, gestational hypertension, and postpartum hemorrhage. These risks are mitigated by appropriate levothyroxine treatment. Levothyroxine/L-triiodothyroxine combinations and desiccated thyroid preparations have the potential to correct maternal hypothyroidism, but the T4 level may still be too low to provide the transplacental delivery necessary for optimal fetal health. The most appropriate pregnancy planning advice is to continue the current dosage of levothyroxine with a plan for monthly monitoring of TSH and T4 during pregnancy, with the expectation that an increase in dosage may be required as the pregnancy progresses.

90. A 55-year-old male has a 3-month history of chronic shortness of breath and dyspnea on exertion. His physical examination is unremarkable except for 1+ ankle edema bilaterally and a soft systolic murmur. A stress echocardiogram is normal. Pulmonary function tests are normal except for a low diffusing capacity of the lung for carbon monoxide (DLCO). Which one of the following conditions should be considered in this patient? F) Chronic pulmonary thromboembolism G) Emphysema H) Interstitial lung disease I) Asthma J) Hypersensitivity pneumonitis

ANSWER: A A diffusion capacity test assesses how well a tracer gas in inspired air can cross from the air into the blood. The diffusion capacity provides a general assessment of the air-blood interface. Reduced values are seen with severe interstitial fibrosis, or when the capillary surface has been compromised by vascular obstruction (pulmonary embolism) or is destroyed by emphysema. Chronic pulmonary embolism causes a low diffusing capacity of the lung for carbon monoxide (DLCO) with normal pulmonary function tests. Emphysema causes a low DLCO and an obstructive pattern on pulmonary function testing (PFT). Interstitial lung disease and hypersensitivity pneumonitis both cause a low DLCO with a restrictive pattern of PFTs. Patients with asthma may have an increased DLCO with an obstructive pattern, with reversibility after bronchodilator administration.

33. The parents of a 4-year-old male bring him in for evaluation because of behavioral problems in his preschool. They report that he is inattentive, hyperactive, and impulsive, has difficulty remaining seated, always seems to be moving, frequently interrupts others, and talks incessantly. His teacher also told them that he never plays quietly, has difficulty taking turns, and intrudes often in other children's play. Which one of the following is recommended by the American Academy of Pediatrics for initial management in this child's case? P) Behavioral treatment alone Q) Methylphenidate (Ritalin) alone R) Atomoxetine (Strattera) alone S) Methylphenidate combined with behavioral treatment T) Methylphenidate combined with atomoxetine

ANSWER: A According to the American Academy of Pediatrics, preschool-age children with ADHD should receive behavioral therapy alone, administered by a parent and/or teacher. Initially prescribing behavioral therapy alone is supported by strong overall evidence and also by a study finding that many preschool-age children with moderate to severe dysfunction had improved symptoms with behavioral therapy alone. If significant improvement is not observed, then methylphenidate can be added. Medications combined with behavioral therapy should be prescribed in elementary school-age children. Evidence for the use of stimulants is strong, and evidence for the use of atomoxetine is sufficient, but not as strong as for the stimulants.

12. A 30-year-old female sees you because of increasing fatigue. She has no chronic medical problems and reports no recent acute illnesses. She recalls being told that she was mildly anemic after the birth of her daughter 3 years ago. The anemia resolved after 3 months of oral iron supplementation. The patient's menstrual periods are regular and last approximately 6 days, with heavy bleeding for the first 3 days then moderate to mild flow for approximately 3 days. She denies epistaxis, black stools, or other signs of bleeding. On examination her temperature is 36.7°C (98.1°F), pulse rate 93 beats/min, respiratory rate 16/min, and blood pressure 116/58 mm Hg. The remainder of her physical examination is unremarkable. A CBC is notable for a hemoglobin level of 10.9 g/dL (N 12.0-16.0) and a mean corpuscular volume of 70 :m3 (N 78-102). Which one of the following serum levels would be most appropriate for further evaluating her microcytic anemia at this point? N) Ferritin O) Folate P) Erythropoietin Q) Hemoglobin A1c R) TSH

ANSWER: A After confirmation of anemia and microcytosis on a CBC, a serum ferritin level is recommended (SOR C). If the ferritin level is consistent with iron deficiency anemia, identifying the underlying cause of the anemia is the priority. A common cause of iron deficiency anemia in premenopausal adult women is menstrual blood loss. If the serum ferritin level is not consistent with iron deficiency anemia, the next stage of the evaluation should include a serum iron level, total iron-binding capacity (TIBC), and transferrin saturation (SOR C). Iron deficiency anemia is still probable if the serum iron level and transferrin saturation are decreased and TIBC is increased. It is more likely anemia of chronic disease if the serum iron level is decreased and the TIBC and transferrin saturation are decreased or normal. Other laboratory tests that may help in differentiating the cause of microcytosis include hemoglobin electrophoresis, a reticulocyte count, and peripheral blood smears.

35. A 50-year-old female with a history of refractory hypertension presents with abdominal pain. Her laboratory results are significant for a positive Helicobacter pylori antibody. You decide to initiate treatment for her H. pylori infection with sequential therapy using the following drug regimen: rabeprazole (Aciphex) plus amoxicillin, followed by clarithromycin (Biaxin) plus tinidazole (Tindamax). She is currently on multiple medications for her hypertension. Which one of her antihypertensive agents would be most affected by the treatment regimen described? E) Amlodipine (Norvasc) F) Clonidine transdermal (Catapres-TTS) G) Hydrochlorothiazide H) Metoprolol tartrate (Lopressor) I) Ramipril (Altace)

ANSWER: A Amlodipine is metabolized by the cytochrome P450 3A4 enzyme. Clarithromycin is a strong 3A4 inhibitor that can slow the metabolism of calcium channel blockers metabolized by this enzyme, thus increasing their levels. This can lead to hypotension, edema, and acute kidney injury due to decreased renal perfusion. It is preferable to choose a different antibiotic regimen for patients on a dihydropyridine calcium channel blocker such as amlodipine, but if another antibiotic cannot be used, either temporarily stopping the calcium channel blocker or empirically lowering the dosage should be considered.

86. A 2-week-old female is brought to the office for a well child visit. The physical examination is completely normal except for a clunking sensation and feeling of movement when adducting the hip and applying posterior pressure. Which one of the following would be the most appropriate next step? F) Referral for orthopedic consultation G) Reassurance only, and follow-up in 2 weeks H) Triple diapering and follow-up in 2 weeks I) A radiograph of the pelvis

ANSWER: A Developmental dysplasia of the hip encompasses both subluxation and dislocation of the newborn hip, as well as anatomic abnormalities. It is more common in firstborns, females, breech presentations, oligohydramnios, and patients with a family history of developmental dysplasia. Experts are divided as to whether hip subluxation can be merely observed during the newborn period, but if there is any question of a hip problem on examination by 2 weeks of age, the recommendation is to refer to a specialist for further testing and treatment. Studies show that these problems disappear by 1 week of age in 60% of cases, and by 2 months of age in 90% of cases. Triple diapering should not be used because it puts the hip joint in the wrong position and may aggravate the problem. Plain radiographs may be helpful after 4-6 months of age, but prior to that time the ossification centers are too immature to be seen. Because the condition can be difficult to diagnose, and can result in significant problems, the current recommendation is to treat all children with developmental dysplasia of the hip. Closed reduction and immobilization in a Pavlik harness, with ultrasonography of the hip to ensure proper positioning, is the treatment of choice until 6 months of age. The American Academy of Pediatrics recommends ultrasound screening at 6 weeks for breech females, breech males (optional), and females with a positive family history of developmental dysplasia of the hip. Other countries have recommended universal screening, but a review of the literature has not shown that the benefits of early diagnosis through universal screening outweigh the risks and potential problems of overtreating.

37. A 39-year-old female presents with a 4-month history of gradually worsening left elbow pain. She does not recall an injury but frequently lifts and holds her 10-month-old son in her left arm. She has tenderness over the lateral epicondyle. Her elbow range of motion is normal but she has pain with supination and pronation. The remainder of the examination is normal. For long-term pain relief, the best evidence supports which one of the following? O) Expectant/conservative management P) Physical therapy Q) Oral anti-inflammatory agents R) A corticosteroid injection

ANSWER: A Lateral epicondylitis is a common condition characterized by degeneration of the extensor carpi radialis muscle tendon originating in the lateral epicondyle. It is a self-limited condition and usually resolves within 12-18 months without treatment. It is not an inflammatory condition and anti-inflammatory agents have not been found to be beneficial. Corticosteroid injections have been found to be associated with poor long-term outcomes, as well as high recurrence rates. Neither physical therapy, bracing, nor splinting is proven to provide long-term pain relief. Approximately 90%-95% of all patients with lateral epicondylitis show improvement at 1 year despite the type of therapy utilized (SOR A).

45. A 55-year-old overweight male presents with a complaint of pain in the left big toe. He recently started jogging 2 miles a day to try to lose weight, but has not changed his diet and says he drinks 4 cans of beer every night. The pain has developed gradually over the last 2 weeks and is worse after running. An examination shows a normal foot with tenderness and swelling of the medial plantar aspect of the left first metatarsophalangeal joint. Passive dorsiflexion of the toe causes pain in that area. Plantar flexion produces no discomfort, and no numbness can be appreciated. Which one of the following is the most likely diagnosis? E) Sesamoid fracture F) Gout G) Morton's neuroma H) Cellulitis

ANSWER: A Pain involving the big toe is a common problem. The first metatarsophalangeal (MTP) joint has two sesamoid bones, and injuries to these bones account for 12% of big-toe injuries. Overuse, a sharp blow, and sudden dorsiflexion are the most common mechanisms of injury. Gout often involves the first MTP joint, but the onset is sudden, with warmth, redness, and swelling, and pain on movement of the joint is common. Morton's neuroma typically causes numbness involving the digital nerve in the area, and usually is caused by the nerve being pinched between metatarsal heads in the center of the foot. Cellulitis of the foot is common, and can result from inoculation through a subtle crack in the skin. However, there would be redness and swelling, and the process is usually more generalized. Sesamoiditis is often hard to differentiate from a true sesamoid fracture. Radiographs should be obtained, but at times they are nondiagnostic. Fortunately, treatment is similar for both conditions, unless the fracture is open or widely displaced. Limiting weight bearing and flexion to control discomfort is the first step. More complex treatments may be needed if the problem does not resolve in 4-6 weeks.

3. A 52-year-old male has a skin lesion removed from his arm with appropriate sterile precautions. Which one of the following would be most appropriate to use on this surgical wound? A) Petrolatum B) Silver sulfadiazine (Silvadene) cream C) Mupirocin (Bactroban) ointment D) Polymyxin B/bacitracin ointment (Polysporin) E) Triple-antibiotic (neomycin/polymyxin B/bacitracin) ointment

ANSWER: A The American Academy of Dermatology recommends against the routine use of topical antibiotics for clean surgical wounds, based on randomized, controlled trials. Topical antibiotics have not been shown to reduce the rate of infection in clean surgical wounds compared to the use of nonantibiotic ointment or no ointment. Studies have shown that white petrolatum ointment is as effective as antibiotic ointment in postprocedure care. Topical antibiotics can aggravate open wounds, hindering the normal wound-healing process. In addition, there is a significant risk of developing contact dermatitis, as well as a potential for antibiotic resistance. Antibiotic treatment should be reserved for wounds that show signs of infection.

62. An asymptomatic 56-year-old male has an echocardiogram that demonstrates trivial mitral regurgitation. Which one of the following is the recommended follow-up for this patient if he remains asymptomatic? O) No repeat echocardiography P) Repeat echocardiography in 1 year Q) Repeat echocardiography in 2 years R) Repeat echocardiography in 5 years S) Repeat echocardiography in 10 years

ANSWER: A The American Society of Echocardiography recommends that physicians NOT order follow-up or serial echocardiograms for surveillance after a finding of trace valvular regurgitation on an initial echocardiogram (SOR C). Trace mitral, tricuspid, and pulmonic regurgitation can be detected in 70%-90% of normal individuals and has no adverse clinical implications. The clinical significance of a small amount of aortic regurgitation with an otherwise normal echocardiographic study is unknown

9. A healthy 24-year-old male presents with a sore throat of 2 days' duration. He reports mild congestion and a dry cough. On examination his temperature is 37.2°C (99.0°F). His pharynx is red without exudates, and there are no anterior cervical nodes. His tympanic membranes are normal, and his chest is clear. Which one of the following would be most appropriate at this point? A) Analgesics and supportive care only B) A rapid strep test C) A throat culture and empiric treatment with penicillin D) Azithromycin (Zithromax)

ANSWER: A The Centers for Disease Control and Prevention (CDC) assembled a panel of national health experts to develop evidence-based guidelines for evaluating and treating adults with acute respiratory disease. According to these guidelines, the most reliable clinical predictors of streptococcal pharyngitis are the Centor criteria. These include tonsillar exudates, tender anterior cervical lymphadenopathy, absence of cough, and history of fever. The presence of three or four of these criteria has a positive predictive value of 40%-60%, and the absence of three or four of these criteria has a negative predictive value of 80%. Patients with four positive criteria should be treated with antibiotics, those with three positive criteria should be tested and treated if positive, and those with 0-1 positive criteria should be treated with analgesics and supportive care only. This patient has only one of the Centor criteria, and should therefore not be tested or treated with antibiotics.

27. A 75-year-old female is evaluated in the emergency department in the evening for heart failure. She is acutely symptomatic with dyspnea. Vital signs include a pulse rate of 96 beats/min, a blood pressure of 140/90 mm Hg, and an oxygen saturation of 94% on room air. A chest radiograph shows mild pulmonary congestion. Which one of the following would be most appropriate regarding placement of an indwelling urinary catheter for accurate measurement of urine output and for patient comfort? A) Avoiding indwelling urinary catheter placement B) Placement of an indwelling urinary catheter only until initial diuresis is completed C) Placement of an indwelling urinary catheter and removal when the patient is transferred out of the emergency department D) Placement of an indwelling urinary catheter until 6:00 a.m. tomorrow E) Placement of an indwelling urinary catheter and removal within 24 hours

ANSWER: A The Society of Hospital Medicine recommends that urinary catheters not be placed or left in place for managing incontinence or for staff convenience, or for monitoring output in patients who are not critically ill. The Infectious Diseases Society of America recommends using patient weight to monitor diuresis. Acceptable indications for an indwelling catheter include critical illness, obstruction, hospice care, and perioperatively for <2 days for urologic procedures.

40. A 56-year-old male is brought to the emergency department by his wife because of a 3-day history of fever up to 102.1°F (38.9°C). He complains of headache, body aches, and a cough. His wife notes that he seems to be confused at times, and mentions that he has type 2 diabetes mellitus. On examination the patient's temperature is 38.7°C (101.7°F), heart rate 113 beats/min, blood pressure 96/64 mm Hg, respiratory rate 24/min, and oxygen saturation 93% on room air. You administer 2 L of oxygen via nasal cannula and his oxygen saturation rises to 98%. A CBC, blood cultures, and a basic metabolic panel are ordered, as well as a chest radiograph and urinalysis. In addition to starting antibiotics, which one of the following would be most appropriate at this point? J) A bolus of normal saline K) Bicarbonate therapy L) Vasopressin (Pitressin) M) Hydrocortisone intravenously N) Norepinephrine

ANSWER: A This patient exhibits signs of possible sepsis, including fever, altered mental status, tachycardia, and tachypnea. Confirmation of a documented infection would establish the diagnosis, but treatment should be started before the infection is confirmed. Initial management includes respiratory stabilization. This patient responded to oxygen supplementation, but if he had not, mechanical ventilation would be indicated. The next appropriate step is fluid resuscitation. A bolus of intravenous fluids at 20 mL/kg over 30 minutes or less is recommended (SOR A). Vasopressors should be started if a patient does not respond to intravenous fluids as evidenced by an adequate increase in mean arterial pressure and organ perfusion (SOR B). First-line agents include dopamine and norepinephrine. Vasopressin may be added but has not been shown to improve mortality. Bicarbonate therapy is not usually recommended to improve hemodynamic status. Hydrocortisone may be used in patients who do not respond to fluids and vasopressors.

96. An 85-year-old male admitted to the hospital for shortness of breath is diagnosed with terminal lung cancer. He decides he would like to receive home hospice care. Over the course of his hospitalization he becomes increasingly confused and forgets where he is and why he is there. He appears depressed with a flat affect. He repeatedly tries to get out of bed and pulls at his IV line and catheter. Which one of the following medications would be most appropriate for treating these symptoms? M) Haloperidol N) Nortriptyline (Pamelor) O) Pentobarbital (Nembutal) P) Lorazepam (Ativan) Q) Mirtazapine (Remeron)

ANSWER: A This patient is showing signs of delirium, which is common in hospice patients. Delirium should be considered in anyone with disturbances of cognitive function, altered attention, fluctuating consciousness, or acute agitation. The mainstay of management is the diagnosis and treatment of any conditions that may cause delirium. Medications that may cause delirium should be discontinued or reduced if possible. Antipsychotic medications are the drug of choice to improve delirium. Central nervous system depressants such as benzodiazepines and barbiturates should be avoided because they can make delirium worse. Nortriptyline has anticholinergic side effects and can also cause delirium. Mirtazapine would not be helpful for treating delirium.

80. An 18-month-old female with atopic dermatitis is brought to your office. She has recently had a flare-up of her condition that has been slow to resolve, and the mother says the child scratches "constantly" despite daily use of emollients. Which one of the following would be the best treatment? F) A topical mild-potency corticosteroid G) Topical pimecrolimus (Elidel) H) Oral diphenhydramine (Benadryl) I) Oral cetirizine (Zyrtec) J) Probiotics

ANSWER: A Topical corticosteroids are the first-line treatment for atopic dermatitis flare-ups. Topical calcineuron inhibitors such as pimecrolimus are a second-line therapy, but carry a warning of a possible link to lymphomas and skin malignancies and are not recommended for children under 2 years of age. Oral antihistamines are not effective for the pruritus associated with atopic dermatitis. Probiotic use is not supported by available evidence.

26. A 29-year-old female presents with redness of her left eye. She has just returned from a summer beach vacation with her children and woke up with a red eye. Your examination reveals a watery discharge, a hyperemic conjunctiva, and a palpable preauricular lymph node. Her cornea is clear on fluorescein staining. Which one of the following is most appropriate for this patient? J) Reassurance only K) Culture-guided antibiotic therapy L) Quinolone eyedrops M) Corticosteroid/antibiotic eyedrops N) Urgent ophthalmologic referral

ANSWER: A Viruses cause 80% of infectious conjunctivitis cases and viral conjunctivitis usually requires no treatment. Bacterial conjunctivitis is associated with mattering and adherence of the eyelids. Topical antibiotics reduce the duration of bacterial conjunctivitis but have no effect on viral conjunctivitis. Allergic conjunctivitis would be more likely if the patient reported itching. Antibiotics or corticosteroids would not be helpful in this patient, and would not prevent complications. The majority of cases of viral conjunctivitis are caused by adenoviruses, which cause pharyngeal conjunctival fever and epidemic keratoconjunctivitis. Pharyngeal conjunctival fever is characterized by high fever, pharyngitis, and bilateral eye inflammation. Keratoconjunctivitis occurs in epidemics, and is associated with a watery discharge, hyperemia, and ipsilateral lymphadenopathy in >50% of cases.

46. A 49-year-old white female is concerned because she has painful, cold fingertips that sometimes turn white when she is hanging out her laundry. Which one of the following medications has been shown to be useful for this patient's condition? I) Propranolol J) Nifedipine (Procardia) K) Ergotamine/caffeine (Cafergot) L) Cilostazol (Pletal)

ANSWER: B There is no currently approved treatment for Raynaud's disease. However, patients with this disorder reportedly experience subjective symptomatic improvement with dihydropyridine calcium channel antagonists, with nifedipine being the calcium channel blocker of choice. Alpha1-Antagonists such as prazosin or terazosin are also effective. Beta-Blockers can produce arterial insufficiency of the Raynaud type, so propranolol and atenolol are contraindicated. Drugs such as ergotamine preparations can produce cold sensitivity, and should therefore be avoided in patients with Raynaud's disease. Cilostazol is indicated for intermittent claudication but not for Raynaud's disease.

88. Which one of the following strategies for preventing the spread of Clostridium difficile infection has been shown to be most effective? N) Use of alcohol-based hand sanitizer O) Handwashing with soap and water P) Screening health care providers for the carrier state Q) Administration of probiotics to at-risk patients R) Use of N95 masks and negative-pressure rooms

ANSWER: B Clostridium difficile infection (CDI) may be transmitted by direct contact with an infected patient, by contact with a contaminated environment, or by contact with a health care worker with transient hand colonization. Effective prevention efforts are essential to limit the spread from one patient to another in the hospital and other health care settings. Although alcohol-based hand antiseptics have been shown to increase compliance with hand hygiene and reduce the incidence of MRSA and VRE infections, alcohol does not kill the spore form of C. difficile and the use of these antiseptics does not reduce the incidence of CDI. There is insufficient data to support the widespread use of probiotics for prevention of CDI, and there is a potential risk of bloodstream infection with their use. Health care workers rarely become colonized with C. difficile, and screening them has not been shown to affect nosocomial transmission rates. Handwashing with soap and water removes C. difficile from the hands of health care workers and remains the cornerstone of prevention efforts. Additional contact precautions such as the use of gloves and gowns may also be helpful. CDI is not transmitted by the respiratory route, so the use of respiratory isolation techniques is not helpful.

69. A 4-year-old male sees you for pre-kindergarten screening. On corneal light reflex testing, the light reflex in the patient's right eye is in the center of the pupil. In the left eye it is located below the pupil, over the inferior-lateral portion of the iris. This clinical finding is associated with a congenital palsy of which one of the following cranial nerves? P) Third Q) Fourth R) Fifth S) Sixth T) Seventh

ANSWER: B In a corneal light reflex test, the patient's attention is attracted to a target while a light is directed at the eyes. In normally aligned eyes the light reflex will be located in the center of each pupil. In patients with esotropia the reflex will be over the lateral portion of the iris in the affected eye. In exotropia the light reflex is over the medial iris, in hypertropia it is over the inferior iris, and in hypotropia it is over the superior iris. The finding observed in this child, hypertropia, will occur with a congenital palsy involving the superior oblique muscle, which is innervated by the fourth cranial nerve.

81. A 53-year-old white female with chronic hepatitis C is concerned about ulcers in her mouth. She is not currently receiving therapy. Your examination reveals several ulcers involving the buccal mucosa. The patient also points out a number of pruritic, reddish-purple plaques on her wrists, ankles, and back. Laboratory studies are within normal limits except for mildly elevated transaminases. Which one of the following is the most likely diagnosis? K) Behçet's syndrome L) Lichen planus M) Aphthous stomatitis N) Herpetic stomatitis O) HIV infection

ANSWER: B Lichen planus is an idiopathic inflammatory disease affecting the skin and oral mucosa. The characteristic violaceous, polygonal papules may be intensely itchy. There is a significant association between lichen planus and hepatitis C virus infection.

18. An 84-year-old male is walking across the street and has to hurry to avoid oncoming traffic. He suddenly develops extreme pain in his knee and falls down, and has to be carried to the sidewalk. The following day he comes to the emergency department. He is comfortable when placed in a knee immobilizer, but is very tender just above the patella. He can bend his knee but when he tries to straighten his leg it is so weak that he cannot move it at all. Radiographs of the knee are shown below. What is the most likely diagnosis? J) Patellar tendon rupture K) Quadriceps tendon rupture L) Tibial plateau fracture M) Patellar subluxation N) Lumbar radiculopathy

ANSWER: B Quadriceps tendon rupture can be partial or complete. When complete, as in this case, the patient has no ability to straighten the leg actively. A similar pattern is seen with patellar tendon rupture, but in this situation the patella is retracted superiorly by the quadriceps. Quadriceps rupture often produces a sulcus sign, a painful indentation just above the patella. If the patient is not examined soon after the injury, the gap in the quadriceps can fill with blood so that it is no longer palpable. The clinical examination is usually diagnostic for this condition, but this patient's radiograph shows some interesting findings, especially on the lateral view. A small shard of the patella has been pulled off and has migrated superiorly with the quadriceps. The hematoma filling the gap in the quadriceps is the same density as the muscle, but wrinkling of the fascia over the distal quadriceps provides a clue that it is no longer attached to the superior margin of the patella. Tibial plateau fractures are intra-articular, so they produce a large hemarthrosis. They are evident on a radiograph in almost all cases. Pain inhibits movement of the knee, but the extreme weakness evident in this case would not be seen. Patellar subluxation is obvious acutely, when the patella is displaced laterally. More often, the patient comes in after the patella has relocated. Findings then include tenderness along the medial retinaculum, sometimes a joint effusion, and a positive apprehension sign when the patella is pushed gently laterally. Lumbar radiculopathy can cause weakness of the quadriceps if it involves the third lumbar root, but complete paralysis would not occur. Other findings would include lumbar pain radiating to the leg, possibly with paresthesias and fasciculations if there were significant neurologic impairment.

57. Which one of the following is a risk factor for prolonged recovery from a sports-associated concussion? K) Blurred vision L) Headache lasting longer than 60 hours M) Amnesia for the injury N) Loss of consciousness at the time of injury O) Convulsions following the injury

ANSWER: B The majority of symptoms associated with sports-related concussions resolve within 72 hours of injury. However, some concussions result in prolonged recovery periods. Risk factors associated with a prolonged recovery include headaches lasting 60 hours or more, self-reported fatigue or fogginess, and four or more symptoms at the onset of injury (SOR B). Loss of consciousness and amnesia have not been found to be related to recovery time. Convulsions associated with the injury are benign and do not affect prognosis. Nausea is one of the symptoms of concussion, but by itself is not a risk factor for prolonged recovery.

60. A 4-month-old female is brought to your office by her parents for a 3-day history of fever up to 101.7°F (38.7°C). She is fussy and her oral intake is down. She has no rash, no emesis, and no diarrhea. Her urine output is normal. She is in day care 3 days a week. On examination she is alert but fussy. Her rectal temperature is 38.4°C (101.1°F). The examination is otherwise normal and there are no focal findings of infection. The parents are reliable and you choose to manage the baby as an outpatient. Which one of the following tests is most likely to be helpful in this situation? E) A CBC with manual differential F) A urinalysis and urine culture G) A chest radiograph H) C-reactive protein I) A lumbar puncture

ANSWER: B The most common causes of serious bacterial infection in children 3-36 months of age are pneumonia and urinary tract infection. In children without an obvious source of infection, the urinalysis and culture are key tests in the evaluation. A valid urine sample should be obtained in all children under the age of 2 with a fever of unknown source. The sample should be obtained through catheterization or suprapubic aspiration. If the patient is toilet trained a clean-catch urine sample is acceptable (SOR C). C-reactive protein is currently under investigation for its utility in detecting serious infection in young children. It is thought to have a greater predictive value than WBC counts but is not yet standardized for common use. A CBC with differential is most useful in neonates but is not as helpful in older infants for detecting serious infection. It is recommended for hospitalized patients but not for those managed as outpatients (SOR C). A chest radiograph is indicated for children with an abnormal respiratory examination or respiratory symptoms. It is also recommended for children older than 1 month of age with a fever >39°C (102°F) and a WBC count >20,000/mm3. A lumbar puncture is indicated for infants with meningeal signs such as focal neurologic findings, petechiae, or nuchal rigidity.

84. A gynecologist requests a preoperative consultation on your patient, a 38-year-old white gravida 2 para 0 abortus 2 whom you referred for total abdominal hysterectomy for adenomatous endometrial hyperplasia. The patient has been hospitalized twice for deep-vein thrombophlebitis in the past 2 years and had spontaneous second-trimester abortions at the ages of 34 and 36. Routine preoperative blood screening reveals the following: Platelet count. . . . . . . . . . . . . . . . . . . . . . . . . . 189,000/mm3 (N 150,000-400,000) Prothrombin time.. . . . . . . . . . . . . . . . . . . . . . 12.0 sec (N 10.0-12.5) INR. . . . . . . . . . . . . . . . . . . . . . . . . . . . . . . . . 1.1 Activated partial thromboplastin time.. . . . . . 42 sec (N 25-35) There is no family history of bleeding disorders and the evaluation is otherwise completely normal. A repeat activated partial thromboplastin time with a 1:1 mixture of normal plasma does not correct to normal. The most likely diagnosis is A) protein C deficiency B) antiphospholipid antibody syndrome C) factor VIII deficiency (hemophilia A) D) chronic liver disease E) von Willebrand disease

ANSWER: B The most common inhibitor discovered during the evaluation of an elevated aPTT is an antiphospholipid antibody. Antiphospholipid antibody syndrome is characterized by venous or arterial thromboembolism and recurrent spontaneous abortion, often in the second trimester, due to placental infarction. Protein C deficiency is associated with recurrent deep vein thrombophlebitis, but does not cause elevation of aPTT. Hemophilia A is associated with an elevated aPTT which corrects with the addition of normal plasma. In chronic liver disease one would expect an elevation of the prothrombin time also. Von Willebrand disease is not associated with thrombophlebitis or recurrent abortion.

70. A 32-year-old primigravida at 36 weeks gestation complains of headaches. She denies vaginal bleeding, leakage of fluid, and contractions, and the fetus is moving normally. Her blood pressure is 155/100 mm Hg and a urinalysis shows 4+ protein. The rest of her examination is normal and a cervical examination shows 1 cm of dilation, 50% effacement, a soft consistency, anterior position, and -2 vertex station. Results of a preeclampsia panel are all in the normal range. Which one of the following is the most appropriate management for this patient? A) Start labetalol (Trandate) and discharge home on bed rest with close follow-up B) Start magnesium sulfate and induce labor now C) Start magnesium sulfate, administer corticosteroids, and induce labor in 48 hours D) Start magnesium sulfate, lower blood pressure to 140/90 mm Hg, and induce labor at 37 weeks gestation E) Arrange for urgent cesarean section

ANSWER: B This patient likely has severe preeclampsia based on her elevated blood pressure with 4+ protein on her urinalysis. Patients with severe preeclampsia near term should be placed on magnesium sulfate to prevent seizures, and labor should be induced immediately. An urgent cesarean section is not necessary. Corticosteroids have not been shown to improve neonatal outcomes when given after 34 weeks gestation. Elevated blood pressures can be managed with hydralazine and labetalol. Normalizing blood pressure is not recommended, but these drugs should be used when blood pressure is over 160/105 mm Hg.

19. A 58-year-old postmenopausal female presents with a recent onset of painless vaginal bleeding. Her last menses occurred 8 years ago and she has had no bleeding until now. She reports that her Papanicolaou smears have always been normal, with the last one obtained a year ago. A pelvic examination today is normal. Which one of the following management options is the preferred next diagnostic step? O) Colposcopy with endocervical curettage P) Transvaginal ultrasonography Q) Saline infusion sonohysterography R) Hysteroscopy

ANSWER: B Transvaginal ultrasonography is the preferred initial test for a patient with painless postmenopausal bleeding, although endometrial biopsy is an option if transvaginal ultrasonography is not available. Transvaginal ultrasonography showing an endometrial thickness <3-4 mm would essentially rule out endometrial carcinoma (SOR C). An endometrial biopsy is invasive and has low sensitivity for focal lesions. Saline infusion hysterography should be considered if the endometrial thickness is greater than the threshold, or if an adequate measurement cannot be obtained by ultrasonography. If hysterography shows a global process, then a histologic diagnosis can usually be obtained with an endometrial biopsy, but if a focal lesion is present hysteroscopy should be considered as the next diagnostic step. Colposcopy is not indicated given the patient's normal Papanicolaou smear.

39. A 62-year-old female with type 2 diabetes mellitus routinely has fasting blood glucose levels in the 80-100 mg/dL range and her hemoglobin A1c level is 7.8%. She has been diligently monitoring her blood glucose levels and all are acceptable with the exception of elevated bedtime readings. She currently is on insulin glargine (Lantus), 18 U at night. Which one of the following changes would be most appropriate for this patient? E) Adding rapid-acting insulin at breakfast F) Adding rapid-acting insulin at lunch G) Adding rapid-acting insulin at dinner H) Increasing the nightly insulin glargine dose I) Increasing the insulin glargine dosage and giving two-thirds in the morning and one-third at night

ANSWER: C This patient continues to have an elevated hemoglobin A-1c and bedtime hyperglycemia. The addition of a rapid-acting insulin at dinner would be the next step in management. For patients exhibiting blood glucose elevations before dinner, the addition of rapid-acting insulin at lunch is preferred. For patients with elevations before lunch, rapid-acting insulin with breakfast would most likely improve glucose control. Increasing or splitting the insulin glargine would be unlikely to improve management.

72. A 62-year-old male has a 1-month history of intermittent vomiting, early satiety, and a weight loss of 4 kg (9 lb). Initially he had diarrhea but it has resolved. He does not have abdominal pain or bloody stools. He says that over-the-counter famotidine (Pepcid) has relieved the symptoms somewhat. Which one of the following would be most appropriate at this point? K) Abdominal radiographs L) Abdominal ultrasonography M) Esophagogastroduodenoscopy N) Famotidine at a higher dosage O) A proton pump inhibitor

ANSWER: C This patient has red flag findings of older age and weight loss with chronic vomiting and is at risk for a gastrointestinal malignancy. He should be referred for esophagogastroduodenoscopy (EGD). Abdominal ultrasonography or radiographs would not be necessary at this time. A proton pump inhibitor or H2 -blocker can be prescribed but should not delay referral for EGD.

68. According to the Joint Commission's sentinel event program, the most common root cause of serious medical errors is a deficiency of K) competency and credentialing L) staffing M) communication N) leadership O) organization culture

ANSWER: C According to The Joint Commission's sentinel event program, communication problems are the most common root cause of serious medical errors. Strategies such as the implementation of structured and standardized handoffs are an example of improving communication among members of the health care team, thereby helping to reduce errors.

36. A 65-year-old female presents with an 11-mm lesion on her nasolabial fold. You perform a shave biopsy that confirms basal cell carcinoma. Which one of the following would be the most appropriate treatment of this lesion? J) Excision with wide margins K) Electrodesiccation and curettage L) Mohs micrographic surgery M) Cryotherapy N) Imiquimod (Aldara) cream

ANSWER: C Basal cell carcinoma is the most common invasive malignant cutaneous neoplasm in humans. The tumor rarely metastasizes but it can advance by direct extension and can destroy normal tissue. Approximately 85% of all basal cell carcinomas occur on the head and neck, with 25%-30% on the nose. Lesions on the nose, eyelid, chin, jaw, and ear have higher recurrence rates than lesions in other locations. A biopsy is necessary to make a definitive diagnosis prior to treatment. Excision is preferred for larger tumors with well-defined borders, but wide margins are not necessary. It is very difficult to perform this surgery with a primary closure around the nose. For lesions around the nose, especially those >1 cm, Mohs micrographic surgery is the preferred treatment. This is a microscopically controlled technique that facilitates removal of the entire lesion with the least amount of tissue removed. Imiquimod is an immune response modifier that can be used on superficial basal cell carcinomas but should not be used for a site with a high risk of recurrence. Electrodesiccation and curettage is effective for smaller nodular basal cell carcinomas. Cryotherapy is not recommended.

10. Routine vaccination against which one of the following organisms has significantly reduced the risk of bacterial meningitis among young children? E) Borrelia burgdorferi F) Escherichia coli G) Haemophilus influenzae H) Listeria monocytogenes I) Mycoplasma pneumoniae

ANSWER: C Conjugate vaccines against Haemophilus influenzae type b and Streptococcus pneumoniae have been highly effective in reducing the incidence of bacterial meningitis in young children and are now routinely recommended for infants and older patients who fall into appropriate risk groups (SOR B). Escherichia coli and Listeria monocytogenes also cause meningitis in young children, but there is not currently a routine vaccine for these pathogens. Likewise, Borrelia burgdorferi and Mycoplasma pneumoniae can cause aseptic meningitis, but there is no routine vaccine.

58. A 53-year-old obese female presents with left calf swelling and tenderness. Using the Wells criteria you determine that she is at intermediate risk for deep vein thrombosis. Which one of the following is the most appropriate next step in the evaluation and treatment of this patient? P) Anticoagulation Q) D-dimer assessment R) Compression ultrasonography S) Impedance plethysmography T) Contrast venography

ANSWER: C In patients at intermediate to high risk for deep vein thrombosis, compression ultrasonography is the initial diagnostic test of choice. In low-risk patients, a negative D-dimer has a high enough negative predictive value to essentially rule out deep vein thrombosis. It is not sensitive or specific enough for evaluating intermediate-risk patients. Initiating treatment based on pretest probability would be inappropriate. Impedance plethysmography is not readily available, and contrast venography is too invasive to be used routinely.

98. An 85-year-old male smoker presents with a 6-day history of subacute abdominal pain. He reports nausea without vomiting, and no change in stool. His past medical history includes coronary artery disease, peripheral vascular disease, and a cholecystectomy. The physical examination reveals moderate periumbilical tenderness without guarding or rebound. Laboratory Findings WBCs.. . . . . . . . . . . . . . . . . . . . . . . . . . . . . . . 20,000/mm3 (N 4500-10,800) Segmented neutrophils.. . . . . . . . . . . . . . . . 82% Bands. . . . . . . . . . . . . . . . . . . . . . . . . . . . . . 7% Chemistry panel.. . . . . . . . . . . . . . . . . . . . . . . normal Urinalysis.. . . . . . . . . . . . . . . . . . . . . . . . . . . . normal Amylase. . . . . . . . . . . . . . . . . . . . . . . . . . . . . . 180 U/L (N <140) Lipase.. . . . . . . . . . . . . . . . . . . . . . . . . . . . . . . normal Lactic acid. . . . . . . . . . . . . . . . . . . . . . . . . . . . 3.8 mmol/L (N 0.5-2.2) Abdominal CT reveals air within the wall of dilated loops of small bowel. Which one of the following is the most likely diagnosis? A) Acute cholangitis secondary to a common duct stone B) Acute diverticulitis C) Acute mesenteric ischemia D) Acute pancreatitis E) Acute appendicitis

ANSWER: C Mesenteric ischemia presents with pain disproportionate to the findings on examination, often with nausea, vomiting, or diarrhea. Air within the wall of dilated loops of small bowel (pneumatosis intestinalis) and evidence of acidosis also suggest bowel ischemia. Cholangitis most likely would be associated with a more substantial elevation of the amylase and/or lipase levels, as well as elevated bilirubin and/or alkaline phosphatase levels. Pancreatitis would also be associated with higher amylase and/or lipase levels. Acute appendicitis often has a vague presentation in older patients, presenting without fever and not localizing to the right lower quadrant as it does in younger patients. However, the leukocytosis is usually not as dramatic as in this case, there is usually no elevation of the amylase or lipase levels, and imaging does not show air within the small bowel.

93. A 21-year-old male comes to your office for a follow-up visit to discuss pharmacologic treatment for his acne. He has moderate inflammatory acne lesions with comedones and several papules and pustules, but few nodules. Multiple topical antibiotic therapies, in combination with benzoyl peroxide, have been minimally effective. He is currently using just topical benzoyl peroxide. You would like to prescribe an oral agent to add to his regimen. Which one of the following would be the most effective oral medication to start at this time? A) Amoxicillin B) Ciprofloxacin (Cipro) C) Minocycline (Minocin) D) Prednisone

ANSWER: C Oral antibiotics are effective for the treatment of moderate to severe acne (SOR A). Combined treatment with benzoyl peroxide is recommended to reduce the risk of bacterial resistance (SOR C). Amoxicillin and ciprofloxacin are not recommended for acne treatment. Intralesional corticosteroid therapies have been tried for acne treatment, but long-term use of oral corticosteroids is not recommended.

20. A 67-year-old male is admitted to your inpatient service with a week-long acute exacerbation of COPD. He also has hypertension and type 2 diabetes mellitus. After 24 hours of intravenous fluids and intravenous methylprednisolone, he is now tolerating oral intake. Which one of the following corticosteroid regimens is best for this patient at this time? A) Continue intravenous methylprednisolone until his COPD is back to baseline, then switch to oral methylprednisolone for a 14-day total course of treatment B) Switch to oral prednisone for a 14-day total course of treatment, including the initial 24-hour intravenous treatment C) Switch to oral prednisone for 4 more days of treatment D) Use only inhaled corticosteroids by nebulizer E) Discontinue corticosteroid treatment altogether after 24 hours

ANSWER: C Systemic corticosteroid therapy reduces the hospital length of stay in patients with acute COPD exacerbations (SOR A). Oral therapy has been shown to be as effective as the intravenous route in patients who can tolerate oral intake (SOR B). A randomized, controlled trial has demonstrated that 5-day courses of systemic corticosteroid therapy are at least as effective as 14-day courses (SOR A). Inhaled corticosteroids are beneficial in some COPD patients but nebulizers generally do not offer significant advantages over metered-dose inhalers in most patients.

66. An 80-year-old male presents with a 10-day history of intermittent colicky abdominal pain. The pain is low and central and seems to be worse after eating. He has no associated fever or vomiting but does feel nauseated when the pain is present. He says that prior to this episode he had hard stools once or twice a week that were difficult to pass. For the past several days he has had only watery stools, several times a day. On examination there is fullness in his left lower quadrant with nonspecific tenderness diffusely and no guarding or rebound. A urine dipstick is normal. Which one of the following is the most likely diagnosis? A) Viral gastroenteritis B) Acute colitis C) Constipation D) Urinary tract infection E) Nephrolithiasis

ANSWER: C The Rome criteria define constipation as the presence of two or more of the following: straining on defecation, hard stools, incomplete evacuation, or less than three bowel movements per week. This patient has multiple symptoms on this list. The presence of watery bowel movements does not rule out the diagnosis of constipation, as it is common for liquid stool to pass an obstructive source.

94. Based on U.S. Preventive Services Task Force guidelines, screening for lung cancer with low-dose CT of the chest is indicated for which one of the following patients with a 30-pack-year smoking history? E) A 50-year-old current smoker F) An 85-year-old current smoker G) A 60-year-old who quit smoking 10 years ago H) A 75-year-old who quit smoking 20 years ago

ANSWER: C The U.S. Preventive Services Task Force recommends annual screening for lung cancer with low-dose computed tomography (LDCT) in adults 55-80 years of age who have a 30-pack-year smoking history and currently smoke or have quit within the past 15 years. Screening should be discontinued once a person has not smoked for 15 years or develops a health problem that substantially limits life expectancy or the ability or willingness to have lung surgery (B recommendation). The risk of lung cancer increases with age and cumulative exposure to tobacco smoke and decreases with time since quitting smoking. The best evidence comes from the National Lung Screening Trial, which enrolled adults age 55-74 who had at least a 30-pack-year smoking history and were current smokers or had quit in the past 15 years. Screening with LDCT resulted in a 20% reduction in lung-cancer mortality among participants. Stratification of participants according to lung cancer risk showed that screening with LDCT prevented the greatest number of deaths from lung cancer among participants with the highest risk and prevented few deaths in the lowest-risk groups. Smoking cessation remains the most effective way to decrease the mortality and morbidity associated with lung cancer, however.

83. A new serum marker has been developed for the diagnosis of pulmonary embolism. The test has a likelihood ratio of 1. Which one of the following conclusions can be made from this information? F) The test can confirm pulmonary embolism G) The test rules out pulmonary embolism H) The test can neither confirm nor rule out pulmonary embolism I) The likelihood ratio does not determine how well a test performs

ANSWER: C The likelihood ratio (LR) is the ratio of the probability of a specific test result in people who have a particular disease to the probability in people who do not. LRs correspond to the clinical impression of how well a test rules in or rules out a given disease. A test with an LR of 1.0 indicates that it does not change the probability of disease. The higher above 1 the LR is, the more likely it is that the disease is present (an LR >10 is considered good). Conversely, the lower the LR is below 1, the more likely it is that the disease is not present (an LR <0.1 is considered good). Likelihood ratios are alternative statistics for summarizing diagnostic accuracy, and have several particularly powerful properties that make them more useful clinically than other statistics.

59. Which one of the following is recommended with regard to the use of osteoporosis medications in elderly patients? A) Substitution of denosumab (Prolia) for bisphosphonates in patients planning extensive dental work B) Use of denosumab in patients at increased risk for infection C) Use of denosumab rather than bisphosphonates in patients with class III or IV renal dysfunction D) Continuous use of bisphosphonates for 10 years or more

ANSWER: C The use of medications for osteoporosis is associated with various side effects, some of which have only recently been recognized. Denosumab and bisphosphonates have similar, albeit low, risks for jaw osteonecrosis. Bisphosphonates should not be used in patients with a creatinine clearance <35 mL/min/1.73 m2, but denosumab is not cleared by the kidneys and is safe in patients with chronic kidney disease. The use of bisphosphonates for more than 5 years can increase the risk of atypical fractures and a holiday from the drug is recommended after either 3 or 5 years, depending on the drug used.

34. A 56-year-old male with diabetes mellitus and hypertension presents with a 6-month history of generalized pruritus. He reports that he scratches frequently. On examination his skin is dry and scaly. He has multiple linear excoriations and thickened skin on his forearms, legs, and neck. Which one of the following is the most likely cause of his pruritus? A) Contact dermatitis B) Chronic urticaria C) Lichen simplex chronicus D) Scabies

ANSWER: C This patient has lichen simplex chronicus, consisting of lichenified plaques and excoriations that result from excessive scratching. Treatment focuses on stopping the itch-scratch cycle. Topical corticosteroids under an occlusive dressing or intralesional corticosteroids can be helpful. Scabies lesions are small, erythematous papules that are frequently excoriated. Contact dermatitis is usually associated with direct skin exposure to an allergen or irritant and is typically localized to the area of exposure. Chronic urticaria causes a typical circumscribed, raised, erythematous lesion with central pallor.

24. A 7-year-old Hispanic female has a 3-day history of a fever of 104.0°F (40.0°C), muscle aches, vomiting, anorexia, and headache. Over the past 12 hours she has developed a painless maculopapular rash that includes her palms and soles but spares her face, lips, and mouth. She has recently returned from a week at summer camp in Texas. Her pulse rate is 140 beats/min, and her blood pressure is 70/40 mm Hg. Which one of the following is the most likely diagnosis? A) Mucocutaneous lymph node syndrome B) Leptospirosis C) Rocky Mountain spotted fever D) Scarlet fever E) Toxic shock syndrome

ANSWER: C While all of the conditions listed are in the differential diagnosis, the most likely in this patient is Rocky Mountain spotted fever (RMSF) (SOR C). It is transmitted by ticks and occurs throughout the United States, but is primarily found in the South Atlantic and South Central states. It is most common in the summer and with exposure to tall vegetation from activities such as camping, hiking, or gardening. The diagnosis is based on clinical criteria that include fever, hypotension, rash, myalgia, vomiting, and headache (sometimes severe). The rash associated with RMSF usually appears 2-4 days after the onset of fever and begins as small, pink, blanching macules on the ankles, wrists, or forearms that evolve into maculopapules. It can occur anywhere on the body, including the palms and soles, but the face is usually spared. Mucocutaneous lymph node syndrome is a similar condition in children (usually <2 years old), but symptoms include changes in the lips and oral cavity, such as strawberry tongue, redness and cracking of the lips, and erythema of the oropharyngeal mucosa. Leptospirosis is usually accompanied by severe cutaneous hyperesthesia. The patient with scarlet fever usually has prominent pharyngitis and a fine, papular, erythematous rash. Toxic shock syndrome may present in a similar fashion, but usually in postmenarchal females.

51. A 25-year-old female kindergarten teacher comes to your office for evaluation of a cough she has had for 2 weeks. The preceding week she had symptoms of rhinorrhea, mild malaise, low-grade fever, and lacrimation. She reports that episodes of coughing are so severe that vomiting is induced. She was evaluated at a walk-in clinic 1 week ago and was diagnosed with bronchitis. Treatment with hydrocodone cough syrup and amoxicillin has not helped. On examination she has mild rhinorrhea and injected conjunctivae, but her lungs are clear. A chest radiograph is normal and her laboratory results reveal a mild lymphocytosis. Which one of the following is the most appropriate next step in the management of this patient? A) Corticosteroid therapy B) A sputum culture C) A nasopharyngeal culture and polymerase chain reaction testing D) Direct fluorescent antibody testing E) Serologic testing

ANSWER: C Whooping cough has reemerged over the past few years. The initial catarrhal stage is manifested by nonspecific symptoms similar to those of a viral upper respiratory illness. This stage is usually 1-2 weeks in duration, and the patient is highly contagious. The paroxysmal stage is manifested by severe coughing spells that occur in paroxysms and may be followed by the inspiratory whoop (much more likely in children). Post-tussive emesis is another classic sign. There are no characteristic findings on examination other than signs induced by extreme coughing. The CDC recommends both a nasopharyngeal culture and polymerase chain reaction testing to confirm the diagnosis. Serologic testing is useful only in research settings, and direct fluorescent antibody testing is not recommended. Azithromycin should be used as initial therapy, but this is to decrease transmission of the illness and does not improve symptoms.

32. Which one of the following findings on pulmonary function testing is most consistent with restrictive lung disease? K) Reduced FEV1 and a decreased FEV1/FVC ratio L) Reduced FEV1 and a normal FEV1/FVC ratio M) Reduced FEV1 and an increased FEV1/FVC ratio N) Reduced FVC and an increased FEV1/FVC ratio O) Decreased diffusing capacity of the lung for carbon monoxide (DLCO)

ANSWER: D A full set of pulmonary function tests consists of spirometry, helium lung volume measurements, and the measurement of diffusing capacity of the lung for carbon monoxide (DLCO). A bronchodilator challenge will allow assessment of reversible airway obstruction. A methacholine challenge test can also be used to look for airway hyperreactivity. A reduced FVC with either a normal or increased FEV1 /FVC ratio is consistent with restrictive lung disease. There are three basic categories of restrictive lung disease: intrinsic lung disease, chest wall deformities, and neuromuscular disorders. A reduced FEV1 and decreased FEV1/FVC ratio is seen in obstructive lung disease (asthma, COPD). The DLCO is the measure of the diffusion of carbon monoxide across the alveolar-capillary membrane. Reduced values are obtained when interstitial fibrosis is extensive, or when the capillary surface is compromised by vascular obstruction or nonperfusion, or is destroyed (as in emphysema).

99. A 75-year-old male reports that his handwriting seems more "cramped," he has started shuffling more as he walks, and he has been experiencing some difficulty turning over in bed, rising from a chair, and opening jars. He also reports increasing body stiffness and a resting tremor in his hand. Given the stage of his disease, which one of the following options for initial medical management is supported by the best evidence? A) Amantadine B) Bromocriptine (Parlodel) C) Benztropine D) Carbidopa/levodopa (Sinemet) E) Entacapone (Comtan)

ANSWER: D All of the drugs listed are used to treat motor symptoms in patients with Parkinson's disease. However, the best evidence supports the use of carbidopa/levodopa, non-ergot dopamine agonists such as pramipexole or ropinirole, or monoamine oxidase-B inhibitors such as selegiline or rasagiline for initial management of patients with early disease (SOR A).

91. In patients who die from an opioid overdose, a second medication is often present that contributes to the patient's death. Which one of the following additional medications is most likely to be found in conjunction with a fatal opioid overdose? K) Acetaminophen L) Antidepressants M) Antipsychotics N) Benzodiazepines O) Muscle relaxants

ANSWER: D In 2010, opioid analgesics were implicated in 75% of pharmaceutical-related overdose deaths. Benzodiazepines were involved in 30% of these opioid analgesic-related deaths. Conversely, opioids were a factor in 77% of pharmaceutical overdose deaths that involved benzodiazepines. Antidepressants are involved in less than half as many opioid deaths as benzodiazepines. Antipsychotics, acetaminophen, and muscle relaxants are implicated in opioid overdose deaths with far less frequency than benzodiazepines.

38. Azithromycin (Zithromax) is prescribed for a 65-year-old male with coronary artery disease. This drug should be used with caution in this patient due to an increased risk for A) an adverse effect on left ventricular function B) peripheral edema C) elevation of systolic blood pressure D) fatal arrhythmias

ANSWER: D In March of 2013 the FDA issued a safety warning regarding azithromycin and its potential to lead to serious and even fatal arrhythmias, particularly in at-risk patients. Risk factors include hypokalemia, hypomagnesemia, a prolonged QT interval, and the use of certain medications to treat abnormal heart rhythms. The mechanism of action is prolongation of the QT interval, leading to torsades de pointes (level of evidence 2, SOR A). The FDA recommends that physicians consider the risk of torsades de pointes and fatal heart rhythms associated with azithromycin when considering antibiotic treatment options, particularly in patients who are already at risk for cardiovascular events.

13. An elderly male who has an implanted cardioverter-defibrillator is admitted to long-term care. He has several chronic comorbidities, including hypertension, a previous stroke, coronary artery disease, osteoarthritis, advanced chronic systolic heart failure, chronic kidney disease with a calculated glomerular filtration rate of 20 mL/min/1.73 m2, diabetes mellitus, and hypercholesterolemia. The patient's quality of life has declined to the point that he wishes to receive only palliative care. He does not want aggressive treatments, including hospitalization, except for reasons of comfort. He has decided he does not wish to be resuscitated, including CPR or intubation. When considering his goals, and after consultation with the patient and his spouse, which one of the following would be most appropriate for managing his defibrillator? A) Adjust the defibrillator to deliver shocks only for ventricular fibrillation B) Adjust the defibrillator to deliver shocks only for a heart rate >140 beats/min C) Remove the defibrillator generator D) Deactivate the defibrillator E) Make no change to the defibrillator

ANSWER: D It is recommended that an implanted cardioverter-defibrillator be deactivated when it is inconsistent with the care goals of the patient and family. In about one-quarter of patients with an implanted cardioverter-defibrillator, the defibrillator delivers shocks in the weeks preceding death. For patients with advanced irreversible disease, defibrillator shocks rarely prevent death, may be painful, and are distressing to caregivers and family members. Advance care planning discussions should include the option of deactivating the implanted cardioverter-defibrillator when it no longer supports the patient's goals.

16. A previously healthy 16-year-old male presents to your office after having a syncopal episode at the start of track practice. An EKG revealed a QTc of 520 ms. This was confirmed on a subsequent EKG. This finding is associated with which one of the following rhythm abnormalities? A) Sinus arrest B) Third degree atrioventricular block C) Paroxysmal supraventricular tachycardia D) Polymorphic ventricular tachycardia E) Atrial fibrillation with a rapid ventricular response

ANSWER: D Patients with repeated EKGs showing a QTc interval >480 ms with a syncopal episode, or >500 ms in the absence of symptoms, are diagnosed with long QT syndrome if no secondary cause such as medication use is present. This syndrome occurs in 1 in 2000 people and consists of cardiac repolarization defects. It is associated with polymorphic ventricular tachycardia, including torsades de pointes, and sudden cardiac death. It may be treated with beta-blockers and implanted cardioverter defibrillators.

79. A 21-year-old female sees you because of a depressed mood since the birth of her son 2 months ago. She is breastfeeding, and her baby is doing well. She reports no difficulties sleeping, other than what is to be expected when caring for a newborn. She denies any suicidal or homicidal ideation and has never had thoughts about hurting the baby. She has a history of depression 2 years ago that was associated with starting college and feeling very isolated in the dormitory. She began taking sertraline (Zoloft), changed her schedule, and spent more time exercising. Within 6 months her depression resolved and she stopped the medication. She reports this current depression feels worse than her previous depression. Which one of the following would be the most appropriate medication for this patient? A) Amitriptyline B) Diazepam (Valium) C) Phenytoin D) Sertraline E) Zolpidem (Ambien)

ANSWER: D SSRIs are the most commonly used medications for postpartum depression. They have fewer side effects and are considered safer than tricyclic antidepressants, especially in depressed women who may be at increased risk for medication overdose (SOR C). In one study, infant serum levels of sertraline and paroxetine were undetectable. It is also recommended that a woman with postpartum depression be started on a medication that she had taken previously with a good response, unless there is evidence of potential harm to her infant (SOR C). Tricyclic antidepressants are excreted into breast milk and there is some concern regarding potential toxicity to the newborn. Phenytoin, diazepam, and zolpidem are not antidepressants. Phenytoin and diazepam are Category D for use in pregnant women. Diazepam is potentially toxic to infants and can accumulate in breastfed infants, and it is not recommended for lactating women (SOR C). Zolpidem is category B in pregnancy and probably acceptable for use in lactating women if clinically indicated.

85. A 45-year-old female who is a new patient sees you for a well care visit. She requests all screening tests and procedures that are appropriate for her. She is unmarried but has been in a monogamous relationship with a male partner for the past 10 years. She reports that she has never had an abnormal Papanicolaou (Pap) test result, but that when she had a Pap test last year she did not have a test for human papillomavirus (HPV). She also had normal findings on a mammogram 1 year ago. Her previous physician had been seeing her every 3 months to monitor her blood pressure, which has consistently been 135-140 mm Hg systolic and 85-90 mm Hg diastolic. She takes no antihypertensive medication, but has instituted dietary and lifestyle changes. According to the U.S. Preventive Services Task Force, which one of the following screening tests or procedures is now recommended for this patient? A) A bimanual pelvic examination with CA-125 testing to screen for ovarian cancer B) A Pap test with co-testing for HPV C) Screening for Chlamydia D) Screening for diabetes mellitus E) Screening for colorectal cancer

ANSWER: D The U.S. Preventive Services Task Force recommends that asymptomatic adults with sustained blood pressure (treated or untreated) >135/80 mm Hg be screened for type 2 diabetes mellitus. Since this patient was screened 1 year ago for cervical cancer, and has no history of an abnormality, she does not require cytology again for another 2 years. If her Papanicolaou test 1 year ago had been combined with HPV testing the rescreening interval could be extended to 5 years if both were negative. Routine screening for ovarian cancer with bimanual examination, transvaginal ultrasonography, or CA-125 testing is not recommended. Chlamydia screening is recommended for high-risk sexually active women over 25 and for all sexually active women age younger than 25. Colorectal cancer screening should begin at age 50.

17. Which one of the following intravenous agents is the best INITIAL management for hypercalcemic crisis? F) Furosemide G) Pamidronate H) Hydrocortisone I) Saline

ANSWER: D The initial management of hypercalcemic crisis involves volume repletion and hydration. The combination of inadequate fluid intake and the inability of hypercalcemic patients to conserve free water can lead to calcium levels >14-15 mg/dL. Because patients often have a fluid deficiency of 4-5 liters, delivering 1000 mL of normal saline during the first hour, followed by 250-300 mL/hour, may decrease the hypercalcemia to less than critical levels (<13 mg/dL). If the clinical status is not satisfactory after hydration alone, then renal excretion of calcium can be enhanced by saline diuresis using furosemide. Intravenous pamidronate, a bisphosphonate, reduces the hypercalcemia of malignancy and is best used in the semi-acute setting, since calcium levels do not start to fall for 24 hours. Glucocorticoids are useful in the treatment of hypercalcemia associated with certain malignancies (multiple myeloma, leukemia, several lymphomas, and breast cancer) or with vitamin D intoxication. The onset of action, however, takes several days, with the effect lasting days to weeks.

44. One week after a complete and adequate baseline screening colonoscopy, a 51-year-old female with no history of previous health problems visits you to review the pathology report on the biopsy specimen obtained from the solitary 8-mm polyp discovered in her sigmoid colon. The report confirms that this was a hyperplastic polyp. Her family history is negative for colon cancer. Which one of the following is the most appropriate interval for follow-up colonoscopy in this patient? A) 1 year B) 2 years C) 5 years D) 10 years

ANSWER: D There is substantial evidence that small (<10 mm) hyperplastic polyps found in the rectum or sigmoid colon are not neoplastic. Data obtained from numerous studies provides considerable evidence of moderate quality that individuals with no significant findings other than rectal or sigmoid hyperplastic polyps of this size should be included in the same low-risk cohort as those who have an unremarkable colonoscopy. For patients at low risk the recommended interval between screening colonoscopies is 10 years. Reductions in this interval are recommended for patients with one or two small tubular adenomas (5-10 years) or those with three or more tubular adenomas (3 years); the interval for more extensive disease is best individualized but can be as often as annually in unusual cases.

41. A 4-year-old male is brought to your office by his parents who are concerned that he is increasingly "knock-kneed." His uncle required leg braces as a child, and the parents are worried about long-term gait abnormalities. On examination, the patient's knees touch when he stands and there is a 15° valgus angle at the knee. He walks with a stable gait. Which one of the following should you do now? A) Refer to orthopedics for therapeutic osteotomy B) Refer to physical therapy for customized bracing C) Prescribe quadriceps-strengthening exercises D) Provide reassurance to the patient and his family

ANSWER: D This case is consistent with physiologic genu valgus, and the parents should be reassured. Toddlers under 2 years of age typically have a varus angle at the knee (bowlegs). This transitions to physiologic genu valgus, which gradually normalizes by around 6 years of age. As this condition is physiologic, therapies such as surgical intervention, special bracing, and exercise programs are not indicated

65. A 2-year-old female is brought to the urgent care center with a fever and cough. Her symptoms started suddenly a few hours ago with a runny nose and fever to 101°F. On examination the child is crying and has a persistent barking cough but has no stridor or significant respiratory distress. Her lungs are clear to auscultation. Her skin is warm, pink, and well perfused, and her oxygen saturation is 99% on room air. A chest radiograph is normal. Which one of the following treatments has been shown to improve outcomes for this problem? J) Humidified air K) Nebulized albuterol (Proventil, Ventolin) L) Oral azithromycin (Zithromax) M) Oral dexamethasone N) Oxygen therapy

ANSWER: D This patient presents with a typical case of mild to moderate croup. This is a viral infection that results in swelling in the larynx. It rarely is severe enough to cause respiratory collapse or require intubation and must be differentiated from more severe conditions such as epiglottitis, retropharyngeal abscess, or pneumonia. There is no reason to treat this viral infection with an antibiotic. The condition is usually benign and self-limiting, with the worst symptoms occurring at night. Cool and/or humidified air has traditionally been recommended, but studies have not confirmed any significant benefit from these interventions. Since this child is not in respiratory distress and oxygenation is normal, supplemental oxygen therapy is not indicated. Studies have confirmed the benefits of treating croup with a single dose of either an oral or intramuscular corticosteroid. Specifically, dexamethasone is recommended due to its 72-hour length of effect. Inhaled racemic epinephrine has been shown to reduce the need for intubation in cases of moderate to severe croup. Albuterol, however, is not indicated.

95. A 37-year-old male complains of severe headaches that typically involve his right eye, and often cause the eye to tear. The headaches occur at about the same time each day and recur for several days in a row before remitting. He reports that he is currently experiencing a third episode of these headaches. Which one of the following therapies will help prevent future recurrences of this patient's headaches? I) Oxygen J) Sumatriptan (Imitrex) K) Lithium L) Verapamil (Calan, Verelan)

ANSWER: D This patient suffers from cluster headaches. Both verapamil and lithium are the mainstays of treatment for chronic cluster headaches, but of the options listed, only verapamil is indicated for the prevention of cluster headaches, and it is actually the first-line prophylactic agent (SOR A). Oxygen and sumatriptan are first-line abortive therapies for cluster headaches (SOR A).

22. Which one of the following ethnic groups in the United States is at greatest risk for complications from influenza? K) African-American L) Asian-American M) Mexican-American N) Native American O) Scandinavian-American

ANSWER: D While anyone, even previously healthy individuals, may benefit from treatment of symptomatic clinical influenza infection with antiviral agents, not everyone who has been exposed but is asymptomatic requires chemoprophylaxis. However, persons at higher risk for complications from influenza should be considered for preventive treatment. Those at highest risk include children under the age of 2 years, pregnant women (including women less than 2 weeks post partum), adults over the age of 65, the morbidly obese (BMI >40 kg/m2), and Native or Alaskan Americans. If persons at high risk for influenza complications are not treated prophylactically with antiviral agents after exposure, then they should receive prompt treatment as soon as possible after developing signs and symptoms of influenza infection.

78. A 45-year-old female has been admitted to the hospital for an episode of acute diverticulitis. Which one of the following features would most strongly suggest a need for surgical intervention? J) A previous admission for diverticulitis in the last 12 months K) Pain uncontrolled by oral analgesics L) A microperforation seen on CT at the site of the diverticulitis M) A 4-cm simple abscess at the site of the diverticulitis N) The presence of generalized peritonitis

ANSWER: E Acute diverticulitis can be treated using oral antibiotics on an outpatient basis in 90% of cases. In fact, there is good evidence that those with uncomplicated diverticulitis (no signs of abscess, fistula, phlegmon, obstruction, bleeding, or perforation) can be treated without the use of antibiotics, using only bowel rest and close follow-up. Among patients who require hospitalization, it is estimated that <10% of cases will require surgical intervention. Thus, the majority of patients hospitalized with this condition, even those with complicated diverticulitis, will respond well to bowel rest and intravenous antibiotics. Indications for surgery include generalized peritonitis, unconfined perforation, uncontrolled sepsis, an undrainable abscess, and failure of conservative management. CT-guided percutaneous drainage of an accessible abscess is a well-proven treatment to avoid the use of open surgery. Prevention of future episodes of diverticulitis increasingly revolves around the use of daily oral medications. Some experts recommend considering surgery to remove a section of bowel after a patient's third admission for diverticulitis.

14. A 50-year-old male with difficult-to-control hypertension seeks your advice regarding progressive breast enlargement. Your examination reveals bilateral firm, glandular tissue in a concentric mass around the nipple-areola complex. You diagnose gynecomastia. Which one of the following antihypertensive medications is most likely to cause this problem? F) Doxazosin (Cardura) G) Hydrochlorothiazide H) Lisinopril (Prinivil, Zestril) I) Losartan (Cozaar) J) Spironolactone (Aldactone)

ANSWER: E Except for persistent pubertal gynecomastia, medication use and substance use are the most common causes of nonphysiologic gynecomastia. Common medication-related causes include the use of antipsychotic agents, antiretroviral drugs, or prostate cancer therapies. Spironolactone also has a high propensity to cause gynecomastia; other mineralocorticoid receptor antagonists, such as eplerenone, have not been associated with similar effects. Discontinuing the contributing agent often results in regression of breast tissue within 3 months.

61. A 36-year-old male with a history of complex regional pain syndrome has been on oxycodone (OxyContin) for the past 5 years. His pain is well controlled. Which one of the following side effects is most likely to occur with long-term chronic use of opioids? J) Diarrhea K) Sedation L) Hypoalgesia M) Respiratory depression N) Hypogonadism

ANSWER: E Hypogonadism is an often underrecognized and undertreated side effect of long-term opioid therapy. It is more often seen in men and in patients receiving larger doses of opioids, including intrathecally. Typical symptoms include decreased libido, erectile dysfunction, amenorrhea, or fatigue. Constipation is not uncommon in patients on chronic opioid therapy, especially if they are elderly, have limited mobility, or are concurrently using other constipating medications. Sedation can occur in the first few weeks after starting therapy but usually tapers off. Hyperalgesia (not hypoalgesia) and allodynia are other side effects resulting from chronic opioid therapy. Respiratory depression is infrequent (SOR C).

67. Which one of the following therapeutic interventions improves outcomes in adults with acute respiratory distress syndrome (ARDS)? F) Early initiation of antibiotics G) Surfactant therapy H) Pulmonary artery catheterization I) Aggressive intravenous fluid resuscitation J) Starting mechanical ventilation with lower tidal volumes

ANSWER: E In patients with acute respiratory distress syndrome (ARDS), starting mechanical ventilation with lower tidal volumes of 6 mL/kg is superior to starting with traditional tidal volumes of 10-14 mL/kg (SOR A). Conservative fluid therapy is recommended in patients with ARDS, as this is associated with a decrease in the number of days on the ventilator and in the intensive-care unit (SOR B). Pulmonary artery catheters are not recommended for routine management of ARDS (SOR A). Surfactant therapy does not improve mortality in adults with ARDS (SOR A), and antibiotics are not an effective treatment.

21. Which one of the following medications used for anxiety has also been shown to reduce the symptoms of irritable bowel syndrome? F) Buspirone G) Clonazepam (Klonopin) H) Divalproex sodium (Depakote) I) Risperidone (Risperdal) J) Citalopram (Celexa)

ANSWER: E Irritable bowel syndrome (IBS) symptoms improve with several different medications and alternative therapies. Exercise, probiotics, antibiotics, antispasmodics, antidepressants, psychological treatments, and peppermint oil all have evidence that they may improve IBS symptoms (SOR B). A Cochrane review of 15 studies involving 922 patients found a beneficial effect from antidepressants with regard to improvement in pain and overall symptom scores compared to placebo. SSRIs used in these trials included citalopram, fluoxetine, and paroxetine, and tricyclic antidepressants included amitriptyline, desipramine, and imipramine. Buspirone, clonazepam, divalproex sodium, and risperidone have not been shown to be effective for symptom relief in IBS patients.

77. Which one of the following is the best exercise to improve function in older adults living in nursing homes? E) Swimming F) Walking G) Stretching H) Stationary bicycling I) Resistance training

ANSWER: E Many types of exercise programs are beneficial for older adults, including simply walking for 30 minutes three times a week. However, a meta-analysis of progressive resistance training programs in nursing homes showed that there were significant improvements in muscle strength, chair-to-stand time, stair climbing, gait speed, and balance. This is seen even in those with advanced age, disabilities, chronic diseases, or extremely sedentary lifestyles.

8. Mild cognitive impairment is characterized by which one of the following? P) Localized motor dysfunction Q) Impairment in at least one activity of daily living R) Impairment in at least one instrumental activity of daily living S) The presence of the APO E4 allele T) Objective evidence of memory decline

ANSWER: E Mild cognitive impairment is an intermediate stage between normal cognitive function and dementia. Motor function remains normal. The presence of the APO E4 allele is a risk factor, but is not necessary for a diagnosis. Patients have essentially normal functional activities but there is objective evidence of memory impairment, and the patient may express concerns about cognitive decline.

1. A 52-year-old female with morbid obesity is incidentally noted to have mildly elevated AST (SGOT) levels. She does not consume alcohol and denies using recreational drugs. A workup for chronic viral hepatitis and hemochromatosis is negative. Which one of the following is most likely to improve her hepatic condition? A) Pentoxifylline B) Simvastatin (Zocor) C) L-carnitine D) Vitamin E E) Weight loss

ANSWER: E Nonalcoholic fatty liver disease is characterized by the accumulation of fat in hepatocytes. It is associated with insulin resistance, central adiposity, increased BMI, hypertension, and dyslipidemia. An incidentally discovered elevated AST level in the absence of alcohol or drug-induced liver disease strongly suggests the presence of nonalcoholic fatty liver disease. The goal of therapy is to prevent or reverse hepatic injury and fibrosis. Diabetes mellitus, hypertension, dyslipidemia, and other comorbid conditions should be appropriately managed. A healthy diet, weight loss, and exercise are first-line therapeutic measures to reduce insulin resistance in patients with nonalcoholic fatty liver disease. Weight loss has been shown to both normalize AST levels and improve hepatic histology. Vitamin E has been shown to improve AST levels but has no impact on liver histology, and pentoxifylline, simvastatin, and L-carnitine have not been shown to consistently improve either AST levels or liver histology (SOR B).

89. A 55-year-old female has severe sepsis due to pyelonephritis. Her systolic blood pressure remains at 70 mm Hg despite antibiotics and adequate fluid resuscitation. Which one of the following should be considered the vasopressor of first choice for this patient? A) Dopamine B) Epinephrine C) Vasopressin (Pitressin) D) Dobutamine E) Norepinephrine (Levophed)

ANSWER: E Norepinephrine is considered to be the vasopressor of choice in this situation. It has mainly "-adrenergic effects with some Beta-adrenergic effects as well. While "-adrenergic agents increase mean arterial pressure, they decrease cardiac output. Norepinephrine's Beta-adrenergic properties counteract the decrease in cardiac output, so there is an increase in blood pressure with little change in pulse rate or cardiac output. Vasopressin may be useful in patients with cardiac arrest and ventricular fibrillation and may further improve hypotension when used with norepinephrine. Dobutamine is the inotropic agent of choice but has little effect on blood pressure. Epinephrine can increase the incidence of arrhythmias when compared with norepinephrine. Dopamine also increases arrhythmias when compared with norepinephrine and is associated with an increased 28-day death rate.

71. A 75-year-old otherwise healthy white female states that she has passed out three times in the last month during her daily brisk walk. Which one of the following is the most likely cause of her syncope? F) Vasovagal syncope G) Transient ischemic attack H) Orthostatic hypotension I) Atrial myxoma J) Aortic stenosis

ANSWER: E Syncope with exercise is a manifestation of organic heart disease in which cardiac output is fixed and does not rise (or even fall) with exertion. Syncope, commonly occurring with exertion, is reported in up to 42% of patients with severe aortic stenosis. Vasovagal syncope is associated with unpleasant stimuli or physiologic conditions, including sights, sounds, smells, sudden pain, sustained upright posture, heat, hunger, and acute blood loss. Transient ischemic attacks are not related to exertion. Orthostatic hypotension is associated with changing from a sitting or lying position to an upright position. Atrial myxoma is associated with syncope related to changes in position, such as bending, lying down from a seated position, or turning over in bed.

15. A large wooden splinter went deep into the forearm of a 24-year-old male while he was working in a horse barn, and he has required local anesthesia and a small incision to remove it completely. After thorough wound cleansing, you inquire about his tetanus status. He is certain that he received all of his primary childhood vaccines and a "tetanus booster" at age 20, but does not know which vaccine he received. Which one of the following is the best choice for this patient regarding tetanus immunization at this time? K) TT (tetanus toxoid) L) Td (tetanus toxoid with reduced diphtheria) M) Tdap (tetanus toxoid with reduced diphtheria and acellular pertussis) N) TIG (tetanus immune globulin) O) No immunization

ANSWER: E The Advisory Committee on Immunization Practices (ACIP) periodically makes recommendations for routine or postexposure immunization for a number of preventable diseases, including tetanus. Since 2005, the recommendation for tetanus prophylaxis has included coverage not only for diphtheria (Td) but also pertussis, due to waning immunity in the general population. The current recommendation for adults who require a tetanus booster (either as a routine vaccination or as part of treatment for a wound) is to use the pertussis-containing Tdap unless it has been less than 5 years since the last booster in someone who has completed the primary vaccination series. In this scenario, no additional vaccination is needed at this time, since the patient is certain of completing the primary vaccinations and received a tetanus booster within the previous 5 years. Had the interval been longer than 5 years, then a single dose of Tdap would be appropriate unless his previous booster was Tdap. Tetanus immune globulin is recommended in addition to tetanus vaccine for wounds that are tetanus-prone due to contamination and tissue damage in persons with an uncertain primary vaccine history. Plain tetanus toxoid (TT) is usually indicated only when the diphtheria component is contraindicated, which is uncommon.

5. A 36-year-old female presents with a several-week history of polyuria and intense thirst. She currently takes no medications. On examination her blood pressure and pulse rate are normal, and she is clinically euvolemic. Laboratory tests, including serum electrolyte levels, renal function tests, and plasma glucose, are all normal. A urinalysis is significant only for low specific gravity. Her 24-hour urine output is >5 L with low urine osmolality. The most likely cause of this patient's condition is a deficiency of A) angiotensin II B) aldosterone C) renin D) insulin E) arginine vasopressin

ANSWER: E This patient has diabetes insipidus, which is caused by a deficiency in the secretion or renal action of arginine vasopressin (AVP). AVP, also known as antidiuretic hormone, is produced in the posterior pituitary gland and the route of secretion is generally regulated by the osmolality of body fluid stores, including intravascular volume. Its chief action is the concentration of urine in the distal tubules of the kidney. Both low secretion of AVP from the pituitary and reduced antidiuretic action on the kidney can be primary or secondary, and the causes are numerous. Patients with diabetes insipidus present with profound urinary volume, increased frequency of urination, and thirst. The urine is very dilute, with an osmolality <300 mOsm/L. Further workup will help determine the specific type of diabetes insipidus and its cause, which is necessary for appropriate treatment. Low levels of aldosterone, plasma renin activity, or angiotensin would cause abnormal blood pressure, electrolyte levels, and/or renal function. Insulin deficiency results in diabetes mellitus.

42. A 71-year-old female with a history of hypertension and osteoporosis presents to your office for preoperative clearance for upcoming eye surgery. She complains of progressively worsening fatigue over the past 8-10 months. She says she often feels dizzy but denies a history of syncope. Her current medications include alendronate and hydrochlorothiazide. You obtain the EKG shown below as part of her preoperative evaluation. Which one of the following would be most appropriate at this point? E) Clearance for eye surgery with no further evaluation F) An exercise treadmill test G) A 48-hour Holter monitor H) A 7-day event monitor I) Referral to a cardiologist for pacemaker placement

ANSWER: E This patient has documented bradycardia on an EKG and a diagnosis of sick sinus syndrome. She has symptomatic end-organ hypoperfusion resulting from her slow heart rate. In addition to lightheadedness and fatigue, other manifestations can include palpitations, angina, heart failure, oliguria, TIA, or stroke. In a symptomatic patient with documented bradycardia, permanent pacemaker placement is recommended (SOR C). If a patient is symptomatic but bradycardia is not evident on the EKG, prolonged monitoring is recommended with a 48-hour Holter monitor. The next step would be longer monitoring with an event monitor. Evidence of sick sinus syndrome may be seen with exercise treadmill testing. Patients with chronotropic incompetence may be unable to achieve target heart rates with exercise. However, the exercise treadmill test is not standardized to diagnose sick sinus syndrome. Patients with sick sinus syndrome may have problems resulting from anesthesia during surgery. Clearing these patients for surgery may not be in their best interest.

49. A 36-year-old male laborer presents to an urgent care center 5 hours after falling off a ladder. He was 7-8 feet off the ground, and he fell directly on his anterolateral leg as he landed. Weight bearing is painful. Foot pulses are normal, as is a sensorineural examination of the foot and leg. The anterolateral lower leg is quite tender but only slightly swollen, and there is exquisite pain in that area with passive plantar flexion of the great toe. Radiographs of the lower leg and ankle are negative. In addition to ice, elevation, and analgesia, which one of the following would be most appropriate? F) Scheduled oral muscle relaxants G) A 6-day oral corticosteroid taper H) Physical therapy referral for early mobilization and ultrasound therapy I) A short leg splint and non-weight bearing for 5-7 days J) Urgent orthopedic referral for possible fasciotomy

ANSWER: E This patient most likely has acute compartment syndrome and must be urgently evaluated by an orthopedic surgeon. Typically, compartment pressure can be measured using a needle attached to a manometer, and if the pressure is elevated (usually >40 mm Hg) urgent fasciotomy is necessary to prevent muscle necrosis. If the classic "Five Ps" (pain, paresthesia, pallor, pulselessness, and paralysis) are all present, the outcome will most certainly be bad, even limb-threatening. Early identification with a high index of suspicion and urgent referral for fasciotomy is necessary to prevent tragic results. Before the classic findings develop, patients will have tenderness out of proportion to the physical appearance of the injury and, most importantly, severe pain in the involved compartment with passive stretching of the involved muscles. While rest, immobilization, non-weight bearing, and analgesia are all appropriate measures, none of these is sufficient treatment for this urgent problem.

63. A 20-year-old offensive lineman who plays football for the small college in your town presents to your office at midseason with pain in his right groin. He describes it as a burning, aching sensation that gets worse when he coughs or strains during a bowel movement, and when he is required to block opponents or push against the blocking sled in practice. As part of the physical examination, you have the patient stand, and you insert your finger into the inguinal canal and follow the spermatic cord to the internal inguinal ring. When you reach the internal ring the patient reports discomfort. When you ask him to cough and strain the pain increases and you feel an impulse or bulge at the tip of your finger. The remainder of his physical examination is normal. This patient's history and examination findings are most consistent with which one of the following diagnoses? A) Athletic pubalgia (sports hernia) B) Osteitis pubis C) Adductor muscle tendinopathy D) Ilioinguinal nerve entrapment E) Inguinal hernia

ANSWER: E This patient's history, along with the bulge/impulse detected on physical examination when he strained or coughed, is most consistent with the diagnosis of inguinal hernia. A "sports hernia" is not a true hernia, but rather a tearing of tissue fibers. The patient often presents with symptoms consistent with a hernia, but without evidence on physical examination. Pain along the symphysis pubis would suggest osteitis pubis, and pain along the adductor tendons would suggest adductor tendinopathy. Ilioinguinal nerve entrapment syndrome is an abdominal muscular pain syndrome characterized by the clinical triad of muscle-type iliac fossa pain with a characteristic radiation pattern, altered sensory perception in the ilioinguinal nerve cutaneous innervation area, and a well-circumscribed trigger point medial to and below the anterosuperior iliac spine.

56. In a patient with symptoms of thyrotoxicosis and elevated free T4, the presence of thyroid TSH receptor site antibodies would indicate which one of the following as the cause of thyroid gland enlargement? F) Toxic multinodular goiter G) Toxic adenoma H) Hashimoto's (lymphadenoid) thyroiditis I) Subacute (giant cell) thyroiditis J) Graves disease

ANSWER: E When there is a question about the cause of goiter and thyrotoxicosis, the presence of TSH receptor immunoglobulins indicates Graves disease. The prevalence of specific forms of TSH receptor site antibodies can distinguish Graves disease from Hashimoto's disease. Both are autoimmune diseases, but in Graves disease there is a predominance of TSH receptor antibodies. In Hashimoto's disease TSH receptor-blocking antibodies are more predominant. These immunoglobulins tend to disappear with therapy.

55. A decrease in which one of the following could be expected from long-term use of postmenopausal estrogen plus progesterone? A) Ischemic heart disease B) Dementia C) All-cause mortality D) Breast cancer E) Hip fracture

ANSWER: E While hormonal therapy was initially used to treat postmenopausal vasomotor symptoms, it was also believed to prevent disease. Based on retrospective studies, a decrease in ischemic heart disease and dementia was suspected. The Women's Health Initiative trials, first undertaken in the 1990s, showed that this was unfortunately not the case, and that hormone therapy actually increases the risk for coronary heart disease, stroke, breast cancer, gallbladder disease, dementia, and venous thrombosis, particularly in older women. A decrease in fractures was demonstrated, however, along with some other health benefits, such as a reduced risk of endometrial cancer.

29. A 57-year-old female is hospitalized for hypotension. She has stage IV breast cancer with extensive visceral and skeletal metastases. For the past 2 weeks she has had fatigue, nausea, and anorexia. She also reports a 3-lb weight loss during this time. She decided to stop chemotherapy 1 month ago. The patient appears pale with a pulse rate of 78 beats/min and a blood pressure of 82/54 mm Hg. Her physical examination is unremarkable except for lower thoracic spine tenderness on percussion. Laboratory studies reveal a serum sodium level of 132 mEq/L, a potassium level of 5.2 mEq/L, and a hemoglobin level of 10.5 g/L. Chest radiographs reveal scattered pulmonary metastatic lesions. The patient is started on intravenous fluid resuscitation with normal saline. On day 2 her blood pressure continues to remain low despite aggressive fluid replacement. Which one of the following should be administered next to manage her hypotension? A) Broad-spectrum antibiotics B) Dobutamine C) Dopamine D) Hydrocortisone E) Packed RBCs

ANSWER:D Common features of acute adrenal insufficiency include fatigue and lack of energy, weight loss, hypotension, loss of appetite, nausea, and vomiting. Other features such as dry skin, hyperpigmentation, and abdominal pain are seen to varying degrees. Common laboratory findings include electrolyte disturbances, hyponatremia, hyperkalemia, hypercalcemia, azotemia, anemia, and eosinophilia. Patients can also have unexplained hypoglycemia. Patients with advanced-stage cancer (especially of the lung or breast) may develop acute adrenal insufficiency from metastatic infiltration of the adrenal glands. Intravenous hydrocortisone is the treatment of choice in the management of adrenal crisis. For managing hypotension, dopamine is recommended for patients with sepsis, dobutamine for those in cardiogenic shock, and packed RBCs for those with hemorrhagic shock. Broad-spectrum antibiotics are part of the therapy for sepsis, but are not first-line agents for hypotension (SOR B).

82. An 86-year-old female nursing-home resident has type 2 diabetes mellitus, chronic diastolic heart failure, chronic kidney disease, advanced osteoarthritis, hypertension, Alzheimer's disease, and other comorbidities. She requires assistance with dressing, bathing, and feeding. For this patient, the American Geriatrics Society recommends a hemoglobin A1c goal of A) <7.0% B) <8.0% C) <8.5% D) <9.0% E) <9.5%

ANSWER: C The American Geriatrics Society consensus report for considering treatment goals for glycemia recommends a hemoglobin A1c goal of <8.5% for individuals with very complex health problems or poor health. This includes individuals in long-term care and those with end-stage chronic illnesses, moderate to severe cognitive impairment, or more than two activity of daily living (ADL) dependencies. A hemoglobin A1c <8.5% equates to an estimated average glucose level of approximately 200 mg/dL. Looser glycemic targets than this may expose patients to acute risks from glycosuria, dehydration, hyperglycemic hyperosmolar syndrome, and poor wound healing.


Set pelajaran terkait

psych chapter3 : Developing Through the Life Span

View Set

Power and Influence Tactics (ch 6)

View Set

Health and Wellness & Leading Causes of Death

View Set

ATI Gastrointestinal learning system 3.0

View Set

Possible Quiz Questions for A Man For All Seasons

View Set